Immunology SBAs Flashcards

1
Q

A 10-year-old boy is seen by a paediatrician after suffering recurrent chest
infections. His mother reports purulent sputum production and cough for the
previous 2 years. Genetic testing reveals the child has a ΔF508 mutation on
chromosome 7. Which physical barrier to infection is most likely to be affected
by the child’s condition?

A Skin
B Gastric acid
C Mucociliary clearance
D Tears
E Gut flora
A

C Mucociliary Clearance

Physical barriers to infection which form part of the innate immune
system provide initial protection against disease-causing organisms.
Impaired mucociliary clearance (C) may arise secondary to cystic fibrosis,
which is the most likely answer in this scenario. Cystic fibrosis is
an autosomal dominant disease which primarily affects the lungs but
also the pancreas, liver and gastrointestinal system. The most common
mutation is the ΔF508 mutation on chromosome 7, which codes for the
cystic fibrosis transmembrane conductance regulator (CFTR). Defective
sodium and chloride ion transport across epithelial cells leads to the formation
of viscous secretions. In the respiratory tract increased viscous
secretions produced by goblet cells cause damage to the cilia, as well as
diffuse lung injury, which can result in bronchiectasis. The skin (A) is
perhaps the most important physical barrier to infection. Although covered
by normal flora, these bacteria are unable to penetrate the numerous
layers which make up the skin. However, severe burns which break
down this important barrier to infection may allow bacteria to enter the
body. Small breaks in the skin that allow a small number of pathogens
to enter the body are usually dealt with by other components of the
innate immune system. The low pH of gastric acid (B) produced in the
stomach destroys most bacteria present in food. Bacteria that reach the
large intestines must compete with commensal gut flora (E); extrinsic
bacteria are therefore unable to replicate and cannot survive. Tears (D)
are produced by the lacrimal glands of the eyes. The lysozyme component
reduces the risk of pathogens entering the eye. Keratoconjunctivitis
sicca (‘dry eye’) is a condition that causes reduced production of tears,
subsequently increasing the risk of infection.

How well did you know this?
1
Not at all
2
3
4
5
Perfectly
2
Q

A 62-year-old woman sees her GP for a regular check-up. On examination, she
has notable deformities of her hands, including swan-neck and Boutonniere
deformities of her fingers. Blood tests reveal a raised CRP. Which of the following
investigation results will most likely feature?

A Reduced AH50 and normal CH50
B Reduced C1 inhibitor
C Reduced C3 and C4
D Reduced C3 and normal C4
E High CH50
A

E High CH50

The complement system is composed of the classical, lectin and alternative
pathways. These individual pathways culminate in the formation of
the membrane attack complex (MAC), which traverses cell surface membranes
of pathogens, causing cell lysis. Components of the complement
system can be quantified in order to differentiate possible diagnoses.
CH50 (total complement activity) measures the level of factors of the
classical and final pathways (C1–C9). As complement factors are acute
phase proteins, a high CH50 (E) indicates acute or chronic inflammation.
Together with the raised CRP and clinical features, this patient is likely
to suffer from rheumatoid arthritis. Systemic lupus erythematosus (SLE)
is a systemic autoimmune disease characterized by antibody-immune
Book Interior Layout.indb 191 28/11/12 7:20 PM
192 Section 6: Immunology SBAs
complex formation and deposition. The classical complement pathway
is composed of C1, C2 and C4. Reduced C3 and C4 (C) levels are typical
of SLE as a result of complex formation (hence consumption) in
an attempt to eliminate immune complexes. C3 and C4 may also be
reduced in SLE due to immunodeficiency which predisposes to developing
the disease. In membranoproliferative glomerulonephritis (MPGN),
anti-nephritic antibodies cause consumption of complement factors,
especially C3. As a result, complement profiling reveals a reduced C3
but normal C4 (D); MPGN type III reflects this pattern particularly well.
AH50 is a laboratory investigation to test for abnormalities of the alternative
pathway, which involves factors C3, B, D and P. A reduced AH50
and normal CH50 (A) suggest possible deficiency of one or more of the
alternative pathway factors; this predisposes to infection by encapsulated
bacteria.
Reduced C1 inhibitor (B) levels indicate hereditary angioedema, characterized
by facial swelling; in severe cases the airway can become compromised
leading to respiratory distress.

How well did you know this?
1
Not at all
2
3
4
5
Perfectly
3
Q

A 25-year-old woman, who has a history of allergy to nuts, is taken to accident
and emergency after eating a dessert containing peanuts. She has an evident
wheeze with an increased respiratory rate, swelling of her lips and itchy skin.
Which cell of the innate immune system is most likely to be responsible for her
symptoms?

A Natural-killer cells
B Dendritic cells
C Eosinophils
D Mast cells
E Neutrophils
A

D Mast cells

Mast cells (D) are involved in the inflammatory process that occurs in
allergy and anaphylaxis (the diagnosis in this case), but also provide
a protective function against pathogens. Mast cells are activated by
one of three mechanisms: direct injury (toxins or drugs), cross-linking
of IgE receptors or by activated complement proteins. Once activated,
mast cells release granules containing histamine and heparin. Histamine
causes vasodilatation leading to the characteristic features of inflammation
(oedema, warmth and redness of the skin). The ‘flare and wheal’
skin reaction is a feature of histamine release by mast cells. Mast cells
play a role in diseases such as asthma, eczema and allergic rhinitis.
Anaphylaxis is characterized by systemic degranulation of mast cells
leading to life-threatening shock. Natural killer cells (NK cells; A) are
responsible for destroying tumour cells and virus-infected cells. NK cells
are unique in that they have the ability to kill such cells in the absence
of antibodies and major histocompatibility complex. Dendritic cells
(B) are antigen-presenting cells (APCs) involved in bridging the gap
between the innate and adaptive immune response. Once dendritic cells
are activated, they migrate to the lymph nodes to facilitate the adaptive
immune system. Eosinophils (C) protect against parasitic infection. Such
pathogens stimulate release of granule contents into the extracellular
space, which surround the parasite and lead to clearance. Neutrophils (E)
are the most common of the granulocytes. Neutrophils are responsible
for the innate protection against bacterial pathogens. Stored within neutrophils
are a host of bactericidal lysosomes which contain lysozyme,
Book Interior Layout.indb 192 28/11/12 7:20 PM
Answers 193
acid hydrolases and myeloperoxidase. Opsonized pathogens are internalized
by neutrophils forming a phagosome. Lysosomal contents enter the
phagosome leading to respiratory burst and lysis of the pathogen.

How well did you know this?
1
Not at all
2
3
4
5
Perfectly
4
Q

A 35-year-old man develops diarrhoea with fever and malaise 24 hours after eating
a take-away meal. Stool cultures reveal the source of the infection is Salmonella
spp. Which antibody is responsible for protecting against gastrointestinal infections?

A IgA
B IgD
C IgG
D IgM
E IgE
A

A IgA

Antibodies (also known as immunoglobulins) are glycoproteins produced
by B cells as part of the adaptive immune system. The basic
role of antibodies is to bind to foreign targets, otherwise known as
antigens. Antibody functions are numerous and include host defence
against pathogens by neutralizing toxins or targeting infective organisms,
complement activation and mast cell stimulation. As well as the
physiological role of antibodies, they are also used in the diagnosis of
infectious diseases by measuring anti-viral and anti-bacterial antibodies.
Structurally, antibodies are made up of two heavy chains and two light
chains. Each heavy chain and each light chain has a constant region
as well as a variable region; the variable regions differ significantly
between antibodies and it is this segment that makes antibodies specific
to target antigens.
IgA (A) can exist as a monomer or a dimer (joined by a short peptide
known as the J chain). Its role is primarily related to the protection of
mucosal surfaces via salivary, respiratory, gastrointestinal and lacrimal
secretions. IgA is also present in breast milk, providing passive immunity
in neonates. IgD (B) is an uncommon immunoglobulin in the body
and is found on the cell surface of immature B cells. IgD provides an
essential role in lymphocyte activation. IgG (C) is the most abundant
antibody and occurs in monomer form in the circulation. The various
subclasses of IgG perform different functions, for example IgG2
is important in fighting encapsulated bacteria. IgG also has a role in
activating complement proteins. IgM (D) occurs as a pentamer and has
a role in the primary response against pathogens. IgE (E) is produced in
response to parasitic infections, as well as during type I hypersensitivity
reactions where it is involved in mast cell activation.

How well did you know this?
1
Not at all
2
3
4
5
Perfectly
5
Q

A 23-year-old man presents to his GP with recent onset diarrhoea, fatigue and
weight loss. The patient suggests that his symptoms are worsened after eating
bread or rice. Which human leukocyte antigen is most likely to be associated
with his disease process?

A HLA B27
B HLA DR2
C HLA DR3
D HLA DR4
E HLA DQ2
A

E HLA DQ2

The human major histocompatibility complex (MHC), otherwise
known as human leukocyte antigen (HLA) system, is the collection
of genes that relates to immune system function and is located
on chromosome 6. The HLA system consists of three major classes:
class I (HLA A, B and C), class II (HLA DP, DQ and DR) and class III
(complement components). HLAs have a number of roles in immunology
including defence against pathogens, transplant rejection and
autoimmune disease.

HLA DQ2 (E) represents a risk factor for coeliac disease (HLA DQ8
is also a risk factor but to a lesser extent). The cell surface receptors
formed by HLA DQ2 bind with greater affinity to α-gliadin, a protein
present in wheat, barley and rye which is responsible for the pathogenesis
of coeliac disease. Therefore, receptors formed from HLA DQ2
are more likely to recruit T cells and initiate an autoimmune response
compared to other HLAs. HLA B27 (A) is associated with ankylosing
spondylitis. The association with HLA B27 suggests the involvement of
CD8+ T cells in the pathogenesis of ankylosing spondylitis. HLA DR2
(B) is associated with Goodpasture’s syndrome, an autoimmune disease
triggered by a type II hypersensitivity reaction. It is characterized by
glomerulonephritis and haemoptysis. HLA DR3 (C) is associated with
Graves’ disease, systemic lupus erythematosus (SLE) and myasthenia
gravis. HLA DR4 (D) is associated with type I diabetes mellitus and
rheumatoid arthritis; in these diseases, HLA DR4 recruits T cells with
subsequent production of islet cell antibodies.

How well did you know this?
1
Not at all
2
3
4
5
Perfectly
6
Q

A 3-year-old Afro-Caribbean boy is referred to a paediatrician after concerns
about his recurrent chest infections. The child’s hair slowly fell out and there is
evidence of depigmentation of his skin. Blood tests reveal hypocalcaemia and
high TSH levels. Which component of the immune tolerance system is likely to
be dysfunctional?

A Regulatory T cell
B TGF-β
C Autoimmune regulator
D Dendritic cells
E IL-10
A

C Autoimmune regulator

T-cell tolerance is the process by which the body’s T cells do not attack
self antigens. There are several mechanisms by which this is achieved,
including the selection of answers given above. Autoimmune disease is
defined as the abnormal response to healthy self components; there is
an underlying pathological process which leads to the breakdown of self
tolerance. Autoimmune disease may be organ specific (Graves’ disease)
or non-organ specific (systemic lupus erythematosus).
Central tolerance is the induction of tolerance to self, which is integrated
into T-cell development in the thymus, a major site for the maturation
of T cells. Within the thymus, T-cell receptors are exposed to
self major histocompatibility complexes (MHC). Those binding to these
MHCs with some affinity are positively selected, whereas those with no
affinity (unable to recognize MHC) are neglected and removed. T cells
binding with high affinity are removed by apoptosis, as these cells pose
an autoimmune risk. The autoimmune regulator (AIRE; C) is also present
within the thymus and presents T-cell receptors with a range of
organ-specific antigens. If T-cell receptors bind to such antigens, they
swiftly die via apoptosis. Autoimmune polyendocrine syndrome type 1
(APECED; associated with mild immune deficiency, dysfunctional parathyroid
gland/adrenal gland, hypothyroidism, gonadal failure, alopecia
and vitiligo) results from mutations in the AIRE gene. The child in this
scenario has features of APECED.
The mechanisms of central tolerance are, however, not fail-safe, and
so peripheral systems exist to remove potential auto-reactive T cells.
Regulatory T cells (A) mature in the thymus and are those that express
CD4, CD25 and Foxp3 on the cell surface. Abnormal Foxp3 leads to the
development of immunodysregulation polyendocrinopathy enteropathy
X-linked syndrome. TGF-β (B) is key in the differentiation of regulatory
T cells, while IL-10 (E) has been found to be expressed by regulatory
T cells; TGF-β and IL-10 are considered to be anti-proliferative and
anti-inflammatory signalling molecules. Dendritic cells (D) can present
peripheral T cells with self antigens. Those T cells which react are killed.
Aberrant dendritic cells have been linked to the development of autoimmune
disease.

How well did you know this?
1
Not at all
2
3
4
5
Perfectly
7
Q

A 34-year-old man presents to his GP with fever, joint pain and a rash on his
trunk. On examination, a new murmur is auscultated. Blood investigations reveal
a raised anti-streptolysin O titre. What is the most likely mechanism for this
disease process?

A Defective immunoregulation
B Molecular mimicry
C T-cell bypass
D Release of hidden self antigens
E Cytokines
A

B Molecular mimicry

Several mechanisms exist by which autoimmune disease can arise. In
this case, the patient has presented with post-streptococcal rheumatic
fever, for which the pathological mechanism is molecular mimicry
(B). Molecular mimicry is the term used to describe the phenomenon
whereby pathogens produce antigens that are molecularly very similar
to self antigens. The immune response to this pathogenic antigen generates
T cells and B cells which are both anti-pathogen and anti-self;
this process is known as immunological cross-reactivity. In the case of
post-streptococcal rheumatic fever, antibodies to M-proteins present on
the surface of group A streptococci cross-react with cardiac myosin; this
results in the inflammatory features of rheumatic fever (fever, raised
ESR/CRP, leukocytosis, carditis). Defective immunoregulation (A) results
in the reduced number or aberrant function of regulatory T cells which
bear CD4, CD25 and Foxp3 surface markers. These cells are responsible
for maintaining peripheral tolerance. Defective immunoregulation has
been associated with thyroid, islet cell and liver autoimmune diseases.
T-cell bypass (B) involves the generation of a novel autoantigen epitope.
Autoantigens are physiologically internalized by B cells, which are in
turn presented to T-helper cells; the B cell is suppressed from producing
autoantibodies. If the complex autoantigen is modified, a new epitope is
provided for T cells to stimulate antibody production by B cells. Triggers
to this modification include drugs and infection, such as Mycoplasma
pneumoniae inducing autoimmune haemolytic anaemia by modifying
erythrocyte surface proteins. Release of ‘hidden’ self antigens (D) may
occur after damage to an organ and causes release of intracellular proteins
which have never been exposed to the immune system. This is the
case post-myocardial infarction, where release of proteins leads to the
generation of autoantibodies against cardiac myocytes (Dressler’s syndrome),
causing pericarditis. Cytokines (E), such as IL-2, may have an
effect on breakdown of immunological tolerance. There is a strong association
between IL-2 therapy (solid-organ tumours) and autoimmune
thyroid disease.

How well did you know this?
1
Not at all
2
3
4
5
Perfectly
8
Q

A 2-year-old girl is seen by an infectious disease paediatrician after suffering
recurrent infections since she was born. Her neutrophil count is normal. A nitroblue-tetrazolium
(NBT) test is performed, which remains colourless. What is the
diagnosis?

A Kostmann syndrome
B Cyclic neutropenia
C Leukocyte adhesion deficiency
D Chronic granulomatous disease
E Von Gierke’s disease
A

D Chronic granulomatous disease

Chronic granulomatous disease (CGD; D) is an X-linked disorder causing
deficiency of NADPH oxidase. As a result, neutrophils cannot produce
the respiratory burst required to clear pathogens. The disease is
characterized by chronic inflammation with non-caseating granulomas.
Clinical features include recurrent skin infections (bacterial) as well
as recurrent fungal infections including Candida spp. and Aspergillus
spp. The disease is usually detected by the age of 5 and is diagnosed
using the nitro-blue-tetrazolium (NBT) test, which remains colourless
due to NADPH deficiency (if NADPH is present the solution turns blue).
The NBT test distinguishes CGD from other phagocyte deficiencies. The
patient will have a normal neutrophil count as there is no defect in
neutrophil production. Treatment involves the use of prophylactic antibiotics
and interferon-gamma. Kostmann syndrome (severe congenital
neutropenia; A) is a congenital neutropenia as a result of failure of
neutrophil maturation. This results in a very low neutrophil count and
no pus formation. NBT test is positive. In leukocyte adhesion deficiency
(LAD; C), neutrophils are formed but cannot exit the blood stream
due to a deficit in leukocyte adhesion molecules resulting in reduced
neutrophil chemotaxis. The neutrophil count is very high due to persistence
in the blood stream. NBT test is positive. Cyclic neutropenia
(B) is an autosomal dominant condition caused by a mutation in the
neutrophil elastase gene (ELA2). Neutropenia occurs every 3 weeks and
lasts approximately 6 days at a time. Cyclic neutropenia improves after
puberty. Von Gierke’s disease (E) is a glycogen storage disease caused
by a deficiency of the enzyme glucose-6-phosphatase. Patients may present
with severe hypoglycaemia. Neutropenia is also a manifestation of
the disease.

How well did you know this?
1
Not at all
2
3
4
5
Perfectly
9
Q

A 29-year-old woman presents to her GP with recent onset joint pain and tiredness.
On examination she has a malar rash. Further blood tests reveal she is antinuclear
antibody and anti-double stranded DNA positive. Which component of
the complement system is she most likely to be deficient in?

A C3
B C4
C C6
D C9
E C1 inhibitor
A

B C4

This patient demonstrates symptoms, signs and diagnostic features consistent
with systemic lupus erythematosus (SLE) and is therefore most
likely to have a deficiency of the classical pathway such as C4 deficiency
(B). Other possible deficiencies in this pathway include C1q, C1r and
C1s and C2. The classical pathway is responsible for clearing immune
complexes and apoptotic cells; patients who have deficiencies in this
pathway therefore have a greater risk of developing immune complex
disease such as SLE. C3 (A) is a common factor in both the classical
and alternative pathways. Deficiency of C3 leads to recurrent pyogenic
infections as there is no C3b (produced via C3 convertase) available to
opsinize bacteria. C3 deficiency also leads to decreased C3a production,
an anaphylatoxin that mediates inflammation. C6 (C) forms part of the
terminal complement pathway, together with C5, C7 and C8, which
form the membrane attack complex (MAC) for bacteriolysis. Deficiency
of terminal complement pathway factors leads to increased susceptibility
to encapsulated bacterial infections, such as Neisseria gonorrhoea
and Neisseria meningitides. While C9 (D) also forms part of the MAC,
patients deficient in C9 still retain some ability to clear encapsulated
bacterial infection, albeit at a slower rate. Therefore, patients deficient
in C9 are usually asymptomatic. C1 inhibitor (E) has the physiological
role of inhibiting the kallikrein system and classical pathway. C1 inhibitor
deficiency causes increased production of bradykinin and spontaneous
activation of the complement pathway; deficiency results in the
autosomal dominant condition hereditary angioedema.

How well did you know this?
1
Not at all
2
3
4
5
Perfectly
10
Q

A 4-year-old girl is seen by a paediatrician to investigate possible developmental
delay and learning difficulties. Initial blood tests reveal hypocalcaemia, reduced
CD4+ and CD8+ T-cell counts as well as deficiency in IgG and IgA. FISH analysis
reveals the child has a deletion of 22q11. What is the diagnosis?

A Di George’s syndrome
B Severe combined immunodeficiency
C Bare lymphocyte syndrome
D Wiskott–Aldrich syndrome
E Interferon-gamma receptor deficiency
A

A Di George’s syndrome

Di George’s syndrome (A) is caused by an embryological abnormality
in the third and fourth branchial arches (pharyngeal pouches) due to a
22q11 deletion. The result is an absent or hypoplastic thymus, as well
as a deficiency in T cells. There is a reduced level or absence of CD4+
and CD8+ T cells as well as decreased production of IgG and IgA. B-cell
and IgM levels are normal. The features of Di George’s syndrome can
be remembered by the mnemonic ‘CATCH’: cardiac abnormalities, atresia
(oesophageal), thymic aplasia, cleft palate and hypocalcaemia. Two
major subtypes of severe combined immunodeficiency (SCID; B) exist,
which affect both T and B cells: X-linked disease (mutation of IL-2
receptor) and an autosomal recessive condition (mutation of adenosine
deaminase gene which leads to a build-up of toxins and hence compromised
proliferation of lymphocytes). Clinical features include diarrhoea,
failure to thrive and skin disease (graft-versus-host induced, caused by
transplacental maternal T cells, and blood transfusion-related caused by
donor T cells). Blood transfusions are contraindicated in patients with
SCID. Bare lymphocyte syndrome (C) is caused by either deficiency in
MHC I (type 1; all T cells become CD4+ T cells) or MHC II (type 2; all
T cells become CD8+ T cells). Clinical manifestations include sclerosing
cholangitis with hepatomegaly and jaundice. Wiskott–Aldrich syndrome
(WAS; D) is an X-linked condition which is caused by a mutation in
the WASp gene which leads to lymphocytopenia. WAS is linked to the
development of lymphomas, thrombocytopenia and eczema. Interferongamma
(IFN-gamma) released by T cells induces the activation of macrophages.
Therefore, IFN-gamma receptor deficiency (E) leads to the
reduced activation of macrophages and so granulomas cannot form,
resulting in increased susceptibility to intracellular infections such as
Mycobacterium tuberculosis and Salmonella spp.

How well did you know this?
1
Not at all
2
3
4
5
Perfectly
11
Q

A 24-year-old man with a history of coeliac disease visits his GP after several
bouts of chest and gastrointestinal infections in the past few years. Although
the infections are mild, the patient is worried about the cause. What is the
diagnosis?

A Severe combined immunodeficiency
B Bruton’s agammaglobulinaemia
C Hyper IgM syndrome
D Selective IgA deficiency
E Common variable immunodeficiency
A

D Selective IgA deficiency

IgA specifically provides mucosal immunity, primarily to the respiratory
and gastrointestinal systems. Selective IgA deficiency (D) results from a
genetic inability to produce IgA and is characterized by recurrent mild
respiratory and gastrointestinal infections. Patients with selective IgA
deficiency are also at risk of anaphylaxis to blood transfusions due to
the presence of donor IgA. This occurs especially after a second transfusion;
antibodies having been created against IgA during the primary
transfusion. Selective IgA deficiency is also linked to autoimmune diseases
such as rheumatoid arthritis, systemic lupus erythematosus and
coeliac disease. The recessive form of severe combined immunodeficiency
(SCID; A) is caused by a mutation of the adenosine deaminase gene
leading to an accumulation of toxins and therefore compromised proliferation
of lymphocytes; CD4+ and CD8+ T-cell levels are decreased.
Reduced proliferation of lymphocytes leads to atrophy of the thymus,
lymph and mucosa-associated lymphoid tissue. Bruton’s agammaglobulinaemia
(B) is an X-linked disease that presents in childhood. It is
caused by a mutation of the BTK gene, which is a tyrosine kinase. This
mutation inhibits B-cell maturation and as a result B-cell and immunoglobulin
levels are diminished. Hyper IgM syndrome (C) is an X-linked
condition that presents in childhood. It is caused by a mutation in
the CD40 ligand on T cells leading to impaired communication with
B cells. B cells are unable to class-switch and therefore only produce
IgM (leading to increased levels in the blood) and patient are deficient
in IgA, IgG and IgE. Patients with hyper IgM syndrome are at risk of
Pneumocystis jerovicci infection. Common variable immunodeficiency
(CVID; E) presents in adulthood. A mutation of MHC III causes aberrant
class switching, increasing the risk of lymphoma and granulomas.
Clinical features include bronchiectasis and sinusitis. Blood tests reveal
a normal IgM level but decreased levels of IgA, IgG and IgE.

How well did you know this?
1
Not at all
2
3
4
5
Perfectly
12
Q

A 40-year-old man is referred to an infectious disease specialist after he is
admitted to hospital with Pneumocystis jerovicci pneumonia. On examination the
patient also has multiple Kaposi’s sarcoma lesions on his chest and abdomen.
What is the most likely diagnosis?

A Inflammatory bowel disease
B Hyposplenism
C Nephrotic syndrome
D AIDS
E Prematurity
A

D AIDS

Broadly, secondary immunodeficiency can result from either reduced
production of immune factors, increased loss or catabolism. Human
immunodeficiency virus (HIV) is a double stranded RNA virus that
causes AIDS (E). AIDS is characterized by immune dysfunction, the
primary defect being a reduced CD4+ T-cell count. AIDS patients
are at greater risk of developing opportunistic infections (for example,
Pneumocystis jerovicci and Cryptosporidium spp.) and tumours
(Kaposi’s sarcoma). Inflammatory bowel disease (IBD; A) is an inflammatory
condition of the gastrointestinal tract that may be subdivided
into ulcerative colitis (UC; affects the colon) and Crohn’s disease (CD;
affects anywhere from the mouth to anus). It is mainly CD that causes
protein losing enteropathy as proteins are absorbed in the small bowel.
The reduced absorption of proteins in IBD results in fewer immunoglobulins
being formed which affects the adaptive immune system
response. Hyposplenism (B) may arise due to splenectomy (after trauma)
or sickle-cell disease, for example. Poor spleen function or absence of
a spleen predisposes to encapsulated bacterial infections, for example
Streptococcus pneumoniae, Haemophilus influenzae and Neisseria meningitidis.
Such patients are therefore required to take necessary vaccinations
and antibiotic prophylaxis. Nephrotic syndrome (C) is characterized
by renal dysfunction leading to large amounts of protein leaking
from the blood to the urine. Consequently, immunoglobulins will be lost
as they are passed into the urine, leading to increased risk of infection
by encapsulated bacteria. Prematurity (E) is a cause of secondary immunodeficiency
as IgG is transferred across the placenta during the final
2 months of pregnancy. Premature babies will have had less IgG transferred
as a fetus. As a result, such babies will be at greater risk of infection
before their own immune systems begin to mature (approximately
4 months after birth).

How well did you know this?
1
Not at all
2
3
4
5
Perfectly
13
Q

A 12-year-old girl has developed a runny nose, itchy eyes and nasal congestion
during the summer months for the past 4 years. She is prescribed anti-histamines
to help her symptoms. Which of the following cells is responsible for the initial
encounter with the allergen?

A Mast cell
B B cell
C Macrophage
D TH1 cell
E TH2 cell
A

C Macrophage

Type I hypersensitivity reactions are mediated by IgE and are associated
with allergy and anaphylaxis. The mechanism behind the development
of type I hypersensitivity reactions begins with the presentation of the
allergen to professional antigen presenting cells. Professional antigen
presenting cells include macrophages (C), dendritic cells and B cells. For
example, if an allergen is taken up by a macrophage, it is processed
intracellularly and peptides are presented via major histocompatibility
complex on the cell surface to T cells of the TH2-cell (E) subclass. TH2-
cell secrete IL-4, which stimulates B cell (B) proliferation. TH1-cells (D)
do not play a role in the pathogenesis of type I hypersensitivity but do
contribute to type IV hypersensitivity reactions. B cells in turn produce
allergen-specific antibodies of the IgE variety. IgE binds to mast cells
(A) via the Fc receptor. During a second exposure, when the allergen
encounters the sensitized mast cell, the surface IgE cross-links which
leads to an increased intracellular calcium concentration, facilitating
the release of pre-formed mediators (histamine, proteases, serotonin and
heparin) as well as newly formed lipid mediators (thromboxane, prostaglandin,
leukotriene and platelet activating factor). These mediators correlate
with the clinical features of allergic reactions. For example, histamine,
leukotrienes and prostaglandins are vasodilators and contribute
to the warmth, oedema and redness which are associated with allergic
inflammation. Examples of diseases caused by type I hypersensitivity
reactions include allergic rhinitis, food allergy and urticaria.

How well did you know this?
1
Not at all
2
3
4
5
Perfectly
14
Q

A 14-year-old girl with a history of eczema presents to accident and emergency
with itching and tingling of her lips and tongue. The girl’s lips are evidently
swollen. All observations are normal. The doctor believes her condition is due to
cross-reactivity of allergens. What is the most likely trigger for her allergy?

A Penicillin
B Eggs
C Nickel
D Dust mite
E Fruit
A

E Fruit

This patient has signs and symptoms confined to her mouth. Together
with the doctor’s suspicions regarding the underlying pathogenesis, oral
allergy syndrome (OAS) is the most likely diagnosis. OAS occurs secondary
to cross-reactivity of antigens inhaled in the mouth, otherwise
known as pollen–food allergy. For example, a patient may be sensitized
to birch pollen; when pollen is breathed in, IgE is created which
cross-reacts with fruit (E) which has been ingested causing release of
histamine from mast cells resulting in local inflammation. Known crossreactants
include birch pollen/stone fruits, mugwort pollen/celery and
ragweed pollen/melon. All symptoms are confined to the mouth only
and include swelling, itching and tingling of the tongue, lips and uvula.
There is often a history of atopic disease. Management includes avoiding
ingestion of the allergen, anti-histamines and prophylactically carrying
an EpiPen in patients who have a history of anaphylaxis. Allergy
to penicillin (A) may result in either acute urticaria or in severe cases,
anaphylaxis. Acute urticaria lasts for less than 6 weeks, characterized
by intermittent rashes which last less than 24 hours at a single site.
Systemic IgE activation results in anaphylaxis characterized by swelling
of the lips, shortness of breath and signs of shock in severe cases.
Eggs (B) are a primary cause of food allergy in children; egg allergy
usually resolves by the age of 8 years. Most food allergies are IgE mediated;
as some are not, the gold standard to test for food allergies is the
double-blind food challenge. Other causes of food allergy include nuts,
shellfish, milk and wheat. Nickel (C) causes contact dermatitis, a type IV
hypersensitivity reaction. A reaction takes 1–2 days to develop (delayed)
leading to desquamation of the skin. As histamines are not involved in
type IV reactions, there is no response to anti-histamines. Dust mites (D)
cause allergic rhinitis, symptomatically characterized by loss of smell,
rhinorrhoea and nasal/eye itchiness.

How well did you know this?
1
Not at all
2
3
4
5
Perfectly
15
Q

A 21-year-old woman is at a Thai restaurant, eating her main course when she suddenly
develops shortness of breath, wheeze and swelling of her lips. The patient
has a known peanut allergy. What is the most appropriate treatment in the first
instance?

A Allergen avoidance
B Adrenaline
C Oral anti-histamines
D Doxepin
E Nasal steroids
A

B Adrenaline

This patient is suffering an anaphylactic attack as a result of peanut
allergy. Other potential causes of anaphylaxis include penicillin, animal
venom and latex. The pathophysiology of anaphylaxis involves
IgE binding to the allergen with subsequent systemic release of histamine
causing vasodilation and contraction of bronchial smooth muscle.
Clinical features include swollen lips, shortness of breath, wheeze and
signs of shock. Anaphylaxis is therefore a medical emergency and intramuscular
(IM) adrenaline (B) is the primary treatment; many patients
who suffer from severe allergy are educated in the use of an EpiPen.
IM adrenaline is the best (and life-saving) choice due to its fast acting
vasocontrictive and bronchodilator effects. Non-IgE mediated systemic
histamine release by mast cells is known as an anaphylactoid reaction.
Causes include opioids, NSAIDs, contrast agents and exercise. Clinical
features are similar to anaphylaxis reaction.
As allergies such as allergic rhinitis, oral allergy syndrome and urticaria
are IgE mediated causing release of histamine by mast cells, oral antihistamines
(C) are the main-stay treatment for such conditions. Antihistamines
used in allergic disease are H1 receptor antagonists which
negate the effects of histamine. Although effective in treating mild
symptoms, oral anti-histamines take longer to have an effect than IM
adrenaline. Nasal steroids (E) may also be prescribed to alleviate symptoms
of rhinorrhoea, itching and nasal congestion. Patients with known
triggers to allergy, such as specific foods, irritants or environmental
conditions, are also encouraged to practice allergen avoidance (A) as a
conservative measure in managing their symptoms. Doxepin (D) is indicated
in the management of chronic urticaria.

How well did you know this?
1
Not at all
2
3
4
5
Perfectly
16
Q

A demanding mother takes her 6-year-old son to see the GP. She is concerned by
his numerous allergies, including pollen and various foods. She is keen for her
son to have allergy testing to determine the substances he is allergic to. Which of
the following would be the best test for investigating allergy in this child?

A Radioallergosorbent test
B Skin prick test
C Double-blind challenge
D Serum tryptase levels
E Total serum IgE
A

B Skin prick test

There are a battery of tests available for the investigation of IgE mediated
hypersensitivity and the triggers which might be causative of such
a reaction. This patient has an allergy to pollen and food: the skin prick
test (B) is the gold standard for investigating such type I hypersensitivity
reactions. The test involves a few drops of purified allergen being
pricked onto the skin. Allergens which are tested for include foods, dust
mites, pollen and dust. A positive test is indicated by wheal formation,
caused by cross-linking of IgE on the mast cell surface leading to
histamine release. Radioallergosorbent test (RAST test; A) is also used
to test for a variety of potential allergens. The test involves patient
serum being added to a range of insoluble allergens. If antibodies are
present to the allergen, these will bind. Radio-labelled anti-human IgE
antibody is then added, which binds to the IgE bound to the insoluble
allergen. Once the unbound IgE is washed away the radioactivity is
measured; the greater the radioactivity the stronger the reaction to the
allergen. Radioactivity-based tests have been replaced by enzyme- and
fluorescence-based assays. The difficulty with RAST testing is that low
IgE levels may be present in the serum which could lead to false negative
results. Double-blind challenges (C) are reserved for food allergies
where there is some doubt after a skin prick or RAST test. This must be
conducted at a centre where necessary equipment is available in case of
anaphylaxis. Serum tryptase levels (D) are useful in diagnosing anaphylaxis
reaction. Measuring total serum IgE (E) is not very informative in
investigating allergy.

How well did you know this?
1
Not at all
2
3
4
5
Perfectly
17
Q

A 56-year-old diabetic man is undergoing a kidney transplant as a result of
chronic renal failure. After the operation, the man immediately develops fever
and has no urine production. Background checks reveal there was an error in
ABO matching of the donor and recipient; the donor’s blood group was A, while
the recipient’s is O. Which of the following immune components is the first to
initiate a response in this case?

A Natural-killer cells
B C1
C Neutrophil
D Mannose binding lectin
E Macrophages
A

B C1

This patient has suffered hyperacute rejection of his graft as a result
of ABO incompatibility; secondary to a previous sensitizing event, the
recipient has developed antibodies that have attacked the allograft. This
is an example of a type II hypersensitivity reaction. Type II hypersensitivity
reactions are IgG and IgM antibody mediated; the antigen is
fixed to tissues or cell surface. Tissue or organ damage is restricted to
those areas where the antibody target exists. Binding of the antibody
to the target antigen causes activation of the classical complement
pathway, beginning with C1 (B); activation of C1 has a number of
effects. Fragments C3a and C5a are subsequently generated and attract
macrophages (E). The final common pathway of complement activation
involves factors C5–C9 forming the membrane attack complex (MAC)
which inserts into the target cell membrane, causing lysis. The classical
pathway also leads to binding of C3b onto the target cell surface membrane,
which causes recruitment of effector cells such as macrophages,
natural-killer cells (NK cells; A) and neutrophils (C). Effector cells cause
significant damage by lysing target cells by an antibody-dependent
cell-mediated cytotoxicity (ADCC) mechanism. Mannose binding lectin
(MBL; D) is part of the lectin complement pathway, which is not
involved in type II hypersensitivity reactions. Further examples of type
II hypersensitivity reactions include myasthenia gravis, pemphigus vulgaris,
haemolytic anaemia and haemolytic disease of the newborn.

How well did you know this?
1
Not at all
2
3
4
5
Perfectly
18
Q

A 54-year-old woman is referred to a dermatologist after developing blisters
which she first noticed in her mouth but have now appeared on her right arm.
On examination, Nikolsky’s sign is positive and immunofluorescent staining
demonstrates the presence of acantholytic cells. What is the most likely target for
antibodies in this case?

A Gastric parietal cell
B Rhesus antigen
C Acetylcholine receptor
D Demoglein 1
E M proteins on group A streptococci
A

D Demoglein 1

Type II hypersensitivity reactions involve the presence of antibodies
that target antigens fixed to the target cell surface membrane. The
patient in question has clinical features of pemphigus vulgaris. Such
features include blistering of the skin and Nikolsky’s sign is positive
(slight rubbing of the skin results in separation of the outermost layer).
Pemphigus vulgaris results from antibodies directed towards demoglein
1 (D) and demoglein 3, which are epidermal cadherins of the epidermis.
Antibodies causing damage to cadherin proteins result in the loss of
linkages between keratinocytes, hence causing the presence of characteristic
acantholytic cells on biopsy. Gastric-parietal cell (A) antibodies
are a feature of pernicious anaemia and lead to parietal cell loss and
hence reduced intrinsic factor production; this causes reduced vitamin
B12 absorption. As a result patients present with vitamin B12 deficiency,
features of which include tiredness (anaemia) as well as sensory or
motor defects. Rhesus antigens (B) are found on the surface of erythrocytes.
As with ABO, the rhesus (Rh) blood group system is a clinically
important system used for matching in blood transfusions. The most
commonly used Rh antigen in matching is the D antigen. Antibodies
directed against the Rh antigen result in autoimmune haemolytic anaemia
(AIHA). The direct Coombs test, which detects antibodies bound
to the surface of erythrocytes, is positive in AIHA. The acetylcholine
receptor (C) located at the neuromuscular junction is the target for autoantibodies
in myasthenia gravis. Myasthenia gravis is a condition which
presents with fatigability of muscles; muscles become fatigued after
periods of movement but recover after rest. In severe cases, muscles of
breathing may become affected, leading to respiratory distress. In poststreptococcal
rheumatic fever, antibodies to M-proteins present on the
surface of group A streptococci (E) cross-react with cardiac myosin; this
results in the inflammatory features of rheumatic fever which include
fever, raised ESR/CRP, leukocytosis and carditis. Rheumatic fever occurs
as a result of molecular mimicry whereby pathogens produce antigens
that are molecularly very similar to self antigens.

How well did you know this?
1
Not at all
2
3
4
5
Perfectly
19
Q

A 35-year-old woman presents to her GP with intermittent fatigue and joint
pain which began 1 month previously. On examination, the patient has a malar
rash on her face. Blood tests reveal anaemia. What is the most likely target for
autoantibodies in this disease process?

A Mouldy hay
B Chlamydia trachomatis
C DNA
D Antiserum proteins
E Hepatitis B virus antigen
A

C DNA

In contrast to type II hypersensitivity reactions, type III hypersensitivity
reactions are characterized by antibodies targeting antigens that are not
fixed to a cell surface. This patient has symptoms and signs characteristic
of systemic lupus erythematosus (SLE). SLE is a multisystem disorder
which may manifest in a number of ways, examples of which include
fever, fatigue, loss of appetite, malar rash, mouth ulcers, photosensitivity,
serositis and joint pains. DNA (C) is the target for circulating antidouble
stranded DNA antibodies in SLE. Many of the clinical features
of SLE result from antibody-immune complex deposition. The presence
of anti-Smith antibodies suggests interstitial lung disease involvement.
Chronic exposure to mouldy hay (A) is the cause of farmer’s lung, an
example of an extrinsic allergic alveolitis. Actinomycetes are the most
common pathogens found in hay dust, which are subsequently inhaled.
Inhalation over prolonged periods of time leads to immune complex
formation as antibodies combine with the inhaled allergen; the immune
complexes are deposited in the walls of the alveoli. Chronic exposure
leads to pulmonary fibrosis, with associated shortness of breath, cyanosis
and cor pulmonale. Antibodies directed at Chlamydia trachomatis
(B) may trigger a reactive arthritis (Reiter’s syndrome). Clinical features
include arthritis, dysuria, conjunctivitis and uveitis. As this phenomenon
is autoimmune, synovial fluid cultures are negative. Proteins in
antiserum (D) are the cause of serum sickness, a self-limiting condition
that occurs when antiserum derived from a non-human animal source
is injected intravenously, resulting in immune complex hypersensitivity.
HBsAg (E) may be associated with the development of polyarteritis
nodosa (PAN), a vasculitis of small and medium sized vessels. Immune
complexes are deposited within such vessels leading to fibrinoid necrosis
and neutrophil infiltration; as a result the vessel walls weaken resulting
in the formation of multiple aneurysms.

How well did you know this?
1
Not at all
2
3
4
5
Perfectly
20
Q

A 34-year-old woman notices an itchy and desquamating, erythematous rash on
her wrist, which has emerged approximately 3 days after wearing a new bracelet.
Which cytokine is the first to be released during the initial exposure to the
allergen?

A IL-10
B IFN-γ
C IL-2
D TNF-α
E IL-12
A

E IL-12

Type IV hypersensitivity (delayed type) reactions are those that are
mediated by T cells of the immune system. These types of reactions
require two exposures to the allergen. During the first encounter,
antigen presenting cells such as macrophages engulf the allergen and
presents peptides on the cell surface via major histocompatibility complex.
CD4+ T cells recognize the peptide and bind to the macrophage.
The macrophage then releases IL-12 (E) which leads to the production
of memory CD4+ T cells of the TH1 variety. During the second exposure,
the macrophage will once again take up the allergen and present
peptide to CD4+ T cells. On this occasion however, the sensitized
memory T cell releases IFN-γ (B), IL-2 (C) and IL-3 thereby activating
macrophages, inducing the production of TNF-α (D); the result is tissue
injury and chronic inflammation. As type IV hypersensitivity reactions
are cell-mediated, there is a lag time of approximately 48–72 hours
before clinical symptoms and signs are visible. IL-10 (E) is not involved
in type IV hypersensitivity reactions; IL-10 is produced by TH2 cells
which causes inhibition of TH1 cells. As a consequence, IFN-γ would
not be produced to activate macrophages and so type IV hypersensitivity
would not occur. An example of a disease process caused by type
IV hypersensitivity is contact dermatitis occurring secondary to nickel
exposure, as is the case in this clinical scenario.

How well did you know this?
1
Not at all
2
3
4
5
Perfectly
21
Q

A 56-year-old woman presents to her GP with blurry vision. On examination the
woman has some bilateral weakness in her legs. The patient mentions that her
vision seems to become more blurry just after she has had a bath. What is the
most likely target in this disease process?

A Pancreatic β-cell proteins
B Nickel
C Proteolipid protein
D Synovial membrane proteins
E Tuberculin
A

D Synovial membrane proteins

Type IV hypersensitivity reactions are mediated by T cells and have
a delayed onset. Proteolipid protein (C) and myelin basic protein are
oligodendrocyte proteins implicated in the pathogenesis of multiple
sclerosis (MS). Multiple sclerosis is a demyelinating disease in which the
myelin sheaths surrounding neurons of the brain and spinal cord are
destroyed. Associated with the disease process is the antigenic stimulation
of CD4+ T cells which in turn activate CD8+ cytotoxic T cells and
macrophages; these are directed at oligodendrocyte proteins, causing
destruction of oligodendrocytes and myelin. Clinical features of MS
include optic neuritis, urinary/bowel incontinence, weakness of the
arms/legs and dysphagia. Uhthoff’s phenomenon describes the worsening
of symptoms that occurs after exposure to higher than ambient
temperatures. Pancreatic β-cell proteins (A) are the antigenic target for
cytotoxic CD8+ T cells in type 1 diabetes mellitus (T1DM). The pathogenesis
involves the destruction of β-cells in the islets of Langerhans in
the pancreas by CD8+ T cells. β-cells are the storage site for insulin in
the body, and so destruction of these cells leads to diminished insulin
release and hyperglycaemia. Presenting features of T1DM include polyuria,
polydipsia and weight loss. Nickel (B) is a hapten and binds with
skin proteins. It is detected by Langerhan’s antigen presenting cells in
the skin causing contact dermatitis. This results in a lesion resembling
eczema with oedema and scaling. Synovial membrane antigens (D) are
the target for T cells in rheumatoid arthritis (RA). RA is defined as a
chronic systemic inflammatory disease causing a systemic, inflammatory
polyarthritis. The Mantoux test involves the intradermal injection
of purified protein derivative tuberculin (E). It is used as a test of previous
exposure to Mycobacterium tuberculosis. A positive test depends
upon a combination of induration size 48–72 hours after the injection,
as well as disease risk factors.

22
Q

A 40-year-old diabetic man is to undergo a kidney transplant as a consequence
of stage 5 chronic kidney disease. The patient has an identical twin who is willing
to donate a kidney, and has been HLA matched at all loci. Which term best
describes the type of organ transplant proposed?

A Autograft
B Split transplant
C Allograft
D Isograft
E Xenograft
A

D Isograft

Transplants of organs are indicated in situations where function is lost
following end-stage disease. In this case the patient in question has
stage 5 chronic kidney disease, also known as end-stage kidney failure,
an irreversible pathology. Transplant is the only cure for such a condition.
The patient has an identical twin (monozygotic) and hence is
genetically very similar. A transplant from the patient’s twin is known
as an isograft (D); as the two individuals will have a similar genetic
profile and the organ has been matched for human leukocyte antigen
(HLA), chance of rejection is rare. Allograft (C) transplants are those
where the donor is of the same species as the recipient but not identical.
As the donor and recipient are genetically different, the organ must
be matched in terms of HLA compatibility, as well as ABO blood group.
HLA and ABO matching minimizes the risk of organ rejection. The use
of immunosuppressive agents is another method of reducing the risk
of transplant rejection. An example of a split transplant (B) is a liver
that may be divided and shared between two recipients. An autograft
(A) is defined as the transplant of tissue to the same patient. Examples
include skin grafts and venous graft for use in coronary artery bypass
graft (CABG) operations. As the tissue or organ is derived from ‘self’
there is zero chance of rejection. A xenograft (E) is defined as an organ
transplant from one species to another. An example of such a transplant
is of a porcine heart valve in an aortic valve replacement which is very
successful. However, in general with xenografts, there is a high risk of
rejection and disease carried in the animal tissue.

23
Q

A 45-year-old man, who has blood group O, has undergone a liver transplant
secondary to chronic alcoholic liver disease which has led to cirrhosis. One hour
after the operation the patient develops a fever and pain in his right upper quadrant.
It is soon realized that the donor had blood group B. Which of the following
best describes the type of allograft rejection?

A Hyperacute rejection
B Acute cellular rejection
C Chronic rejection
D Acute vascular rejection
E Graft-versus-host disease
A

A Hyperacute rejection

Patients who are due to undergo a transplant are matched with a donor
for human leukocyte antigen (HLA) and ABO blood group. In this case,
there has been an error in ABO matching which is a recipe for hyperacute
rejection (A). Hyperacute rejection occurs within minutes to
hours, and is mediated by pre-formed antibodies against antigens on
the surface of the donor organ. The binding of pre-formed antibodies
to the donor organ activates the complement pathway and clotting
cascade, leading to thrombosis and ultimately rejection of the donor
organ. Acute cellular rejection (B) occurs approximately 1 week after
transplantation. It is T-cell mediated (type IV hypersensitivity reaction).
In cases where there is HLA-mismatch, antigen presenting cells present
peptides that are made of foreign HLA to CD4+ T cells, coordinating
an immune response against the donor organ. This involves macrophages,
CD8+ T cells, B-cell and pro-inflammatory cytokines (IFN-γ
and TNF-α) directed towards the donor organ. Acute vascular rejection
(D) may occur with transplant of a xenograft. It is an antibody reaction,
which may either be due to a pre-formed antibody (not detected
at cross-match) or a new antibody produced by activated B cells. The
pathogenesis is similar to hyperacute rejection, but occurs 4–6 days
after transplantation. Chronic rejection (C) involves both immune and
non-immune reactions; it may occur months to years after transplantation.
The pathogenesis involves smooth muscle growth which causes
blockage of graft vessel lumens leading to ischaemia and fibrosis. Risk
factors include HLA-mismatches, multiple acute rejections, hypertension
and hyperlipidaemia. Graft-versus-host disease (GVHD; E) is a complication
of allogeneic stem cell transplantation. It occurs when immune
cells transferred in the donated stem cells recognize the recipient tissue
as foreign causing a graft-versus-host reaction.

24
Q

A 54-year-old man is to undergo a heart transplant as a result of severe heart
failure. Prior to the operation the transplant team initiate an immunosuppressive
regimen using a drug that inhibits calcinurin. Which of the following drugs is
this most likely to be?

A Cyclosporine A
B OKT3
C Azathioprine
D Corticosteroids
E Daclizumab
A

A Cyclosporine A

Cyclosporine A (A) inhibits the protein phosphatase calcineurin. This
causes IL-2 secretion from T cells, a cytokine which stimulates T-cell
proliferation; the production of T cells is reduced, hence minimizing
organ rejection. A common side effect is gum hyperplasia. OKT3
(muromonab-CD3; B) is a mouse monoclonal antibody targeted at the
human CD3 molecule used to treat rejection episodes in patients who
have undergone allograft transplantation. Administration of the antibody
efficiently clears T cells from the recipient’s circulation, T cells
being the major mediator of acute organ rejection. Primary indications
include the acute corticosteroid-resistant rejection of renal, heart
and liver transplants. Anaphylaxis is a major potential adverse effect
of using murine proteins. Azathioprine (C) is an antimetabolite agent
used in immunosuppressive therapy. Azathioprine is metabolized into
6-mercaptopurine (6-MP), a purine analogue that prevents DNA synthesis,
thereby inhibiting the proliferation of cells; lymphocytes are
most affected. Antigen presenting cells present non-self proteins (from
the allograft) to T cells which in turn produce IL-2 to stimulate T-cell
proliferation. However, 6-MP inhibits this proliferation and so reaction
between T cells and the allograft is minimized. Corticosteroids (D) are
used as an immunosuppressive agent in both the prevention and treatment
of transplant rejection. Corticosteroids inhibit phospholipase A2
thereby blocking prostaglandin formation as well as a series of inflammatory
mediators. The immunosuppressive effects of corticosteroids are
numerous and include reducing the number of circulating B cells, inhibiting
monocyte trafficking, inhibiting T-cell proliferation and reducing
the expression of a number of cytokines, for example, IL-1, IL-2 and
TNF-α. Prednisolone is used prophylactically before transplantation to
prevent rejection; methylprednisolone is used in the treatment of rejection.
IL-2 receptor antibody (daclizumab; E) targets the CD25 of IL-2
receptors expressed on the surface of activated T cells. It is especially
used in kidney transplant patients to prevent organ rejection. Common
side effects of all immunosuppressants include increased risk of infections,
hepatotoxicity and malignancy.

25
Q

A 35-year-old man presents to the GP with fever, lymphadenopathy and a sore
throat. Blood tests reveal a leukocytosis and Western blot is positive for HIV
infection. Which of the following proteins is responsible for binding to CD4+
T cells to initiate infection?

A Gag protein
B gp120
C gp41
D Reverse transcriptase
E CCR5
A

B gp120

The human immunodeficiency virus (HIV) is a spherical virus with a
lipid envelope. Risk factors for transmission include anal intercourse,
infected blood products, intravenous drug use and vertical transmission.
Structurally, HIV consists of a core, capsid and envelope. The first
step in HIV infection involves binding of the envelope glycoprotein
gp120 (B) to the CXCR4 receptor on the cell surface of the CD4+ T cell.
Once bound the HIV envelope undergoes structural change allowing
the glycoprotein gp41 (C) to penetrate the CD4+ T-cell wall to stabilize
the attachment. Once bound, HIV can inject viral RNA and replicating
enzymes, including reverse transcriptase (D), integrase and protease, into
the target cell. The RNA undergoes reverse transcription via reverse transcriptase
to form cDNA, which is integrated into the host DNA by integrase.
CD4+ cell death occurs by one of three mechanisms. The infected
CD4+ T cells may be killed by cytotoxic CD8+ T cells; budding of HIV
may cause CD4+ T cells to burst; infected CD4+ T cells may fuse with
uninfected CD4+ T cells forming giant cells (syncytia) that balloon and
die. The Gag protein (A) provides infrastructural support for HIV. HIV
may also bind to macrophages via the cell surface receptor CCR5 (E).
Macrophages infected by HIV are not destroyed but are used as replicating
reservoirs as well as a means of gaining entry to the central nervous
system as macrophages are able to cross the blood–brain barrier. HIV
infection may progress to acquired immunodeficiency syndrome (AIDS)
which is defined by a CD4+ count less than 200cells/µL of blood or an
AIDS-defining illness, for example infection by Mycobacterium avium
intracellulare, Candia albicans oesophagutus and toxoplasmosis. Patients
with AIDS are also at increased risk of developing Kaposi’s sarcoma,
non-Hodgkin’s lymphoma and cervical cancer.

26
Q

A 13-year-old boy is immunized against an acid-fast bacillus species after a
negative Mantoux test. Which term best describes this form of vaccination that
has been administered?

A Live attenuated
B Inactivated
C Subunit
D Conjugated
E Passive immunity
A

A Live attenuated

The boy in this case has had a Mantoux test to determine if he has a
latent tuberculosis (TB) infection. The test is negative and hence he
can be given the Bacillus Calmette–Guèrin (BCG) vaccine to provide
immunity against TB. The BCG vaccine is a live attenuated (A) vaccination,
prepared using a weakened live bovine tuberculosis bacillus. Other
examples of live attenuated vaccines include polio (Sabin), MMR and
typhoid. Live attenuated vaccines provide long-term immunity and protection
against a number of reactive strains; they do not require boosters
or adjuvants as such vaccines trigger a sufficiently strong immune
response. Live attenuated vaccines are contraindicated in immunocompromised
patients. In inactivated (B) vaccines pathogens are destroyed
so they are unable to replicate but retain the ability to induce an
immune response. Examples include vaccines against cholera, hepatitis
A virus and rabies. Inactivated vaccines are suitable for patients with
immunodeficiency but require boosters. Subunit (C) vaccines are characterized
by the use of antigenic proteins (not whole organisms) and
include hepatitis B virus (recombinant), pneumococcal, diphtheria, tetanus
and pertussis vaccines. Conjugated (D) vaccines are those used to
immunize against encapsulated bacteria such as influenza, pneumococcus
and Nissseria meningitides. Passive immunity (E) describes immunity
derived from the transfer of immunoglobulin. This form of immunity
lasts approximately 3 weeks as the immunoglobulin proteins are broken
down within the body. Examples of passive immunity include the use
of human rabies immunoglobulin (HRIg) in rabies cases, as well as prophylactic
and post-exposure use for hepatitis A infection (must be given
within 2 weeks of exposure).

27
Q

A 3-year-old boy is referred to a paediatrician after experiencing recurrent chest
infections. Blood tests demonstrate a reduced B-cell count as well as low IgA,
IgM and IgG levels. Genetic testing reveals a defect in the BTK gene. What is the
best therapeutic modality for this child?

A IFN-α
B IFN-β
C IFN-γ
D Intravenous IgG
E Haematopoietic stem cell transplant
A

D Intravenous IgG

A primary role for immune-based therapies is to boost the immune
response to improve protection against infection and malignancy, especially
in those who are immunodeficient. Bruton’s agammaglobulinaemia
is characterized by a mutation of the BTK gene, a tyrosine kinase. This
mutation leads to inhibition of B-cell maturation and as a consequence
B-cell and immunoglobulin levels are diminished. Blood investigations
will reveal decreased circulating B cells as well as immunoglobulins.
Patients are at risk of recurrent infections, particularly encapsulated
bacteria, and must therefore receive passive immunity to protect against
such pathogens. Intravenous IgG (D) is not a cure for Bruton’s agammaglobulinaemia
but prolongs survival. Treatment must be continued
throughout life. Intravenous IgG is also used in the treatment of hyper
IgM syndrome, common variable immunodeficiency as well as secondary
antibody deficiencies. Haematopoietic stem cell transplant (HSCT; E)
involves the transplant of multipotent haematopietic stem cells which
may either be autologous (from self) or allogenic (from a donor). In
either case, myeloablative techniques are used to destroy the remaining
cells of the bone marrow which leads to increased risk of infection
throughout the course of the treatment. HSCT is indicated in diseases
such as severe combined immunodeficiency (SCID), leukaemia and multiple
myeloma.
Interferons are signalling proteins involved in the immune system in
response to pathogens and tumour cells. They act via the Janus kinaseSTAT
(Jak-STAT) pathway to produce further anti-viral, anti-proliferative
and immunoregulatory factors. IFN-α (A) is used in the treatment of
hepatitis B, hepatitis C, Kaposi’s sarcoma and chronic myeloid leukaemia.
IFN-β (B) is indicated in the treatment of multiple sclerosis, but its mechanism
of action is unknown. IFN-γ (C) is used in the treatment of chronic
granulomatous disease, a disease in which phagocytes lack the enzyme
NADPH, and hence neutrophils are unable to clear pathogens.

28
Q

A 49-year-old woman with known rheumatoid arthritis is seen in the rheumatology
clinic. She has been taking a medication over a long period of
time which is used to control proliferation of her white blood cells. The
patient explains that she has been feeling tired recently and has suffered
with low moods. Routine blood tests reveal she has a macrocytic megaloblastic
anaemia.

A Cyclophosphamide
B Mycophenolate mofetil
C Azathioprine
D Methotrexate
E Cisplatin
A

D Methotrexate

Anti-proliferative agents broadly inhibit DNA synthesis and thereby
interfere with cell proliferation, especially those cells with a high turnover,
for example leukocytes. In this case, the side effects suggestive
of folate deficiency point to methotrexate (D) as the correct answer.
Methotrexate is an anti-metabolite and anti-folate drug indicated for
the treatment of cancer as well as autoimmune diseases including rheumatoid
arthritis and systemic lupus erythematosus. Methotrexate inhibits
dihydrofolate reductase (DHFR), an enzyme involved in the synthesis
of the nucleoside thymidine; thymidine is essential for DNA synthesis.
As folate is required for the synthesis of purine, production of this base
is also disrupted. Ultimately, proliferation of leukocytes is interrupted.
Side effects include those of folate deficiency (macrocytic megaloblastic
anaemia, loss of appetite, tiredness, weakness and depression). The
low white cell count that results predisposes to infection; this is an
adverse effect of all anti-proliferative drugs. Cyclophosphamide (A) is
an alkylating agent, attaching an alkyl group to the guanine base of
DNA. This causes damage to the DNA structure and therefore prevents
cell replication; cyclophosphamide affects B-cell replication more than
T cells. Complications of therapy include bone marrow suppression,
hair loss and carcinogenic properties that may cause transitional cell
carcinoma of the bladder. Mycophenolate mofetil (B) is the pro-drug of
mycophenolic acid which inhibits inosine monophosphate dehydrogenase
(IMPDH), an enzyme required in guanine synthesis; impaired guanine
synthesis reduces the proliferation of both T and B cells, but T cells
are affected to a greater extent. Side effects include bone marrow suppression
(particularly low white blood cells and platelets). Azathioprine
(C) is metabolized in the liver to 6-mercaptopurine which causes the
inhibition of purine synthesis and preferentially inhibits T-cell activation
and proliferation. A proportion of the population have a thiopurine
methyltransferase (TPMT) polymorphism, rendering them unable
to metabolize azathioprine; patients therefore have a predisposition to
azathioprine toxicity. Cisplatin (E) is a chemotherapeutic agent which
cross-links with DNA and interferes with cell proliferation. Side effects
include nephrotoxicity, neurotoxicity and ototoxicity.

29
Q

A 45-year-old man, who suffers from myasthenia gravis’ presents to accident
and emergency with difficulty in breathing. Assisted ventilation is administered.
Which of the following is the best option for the initial management of the
patient’s condition?

A Ciclosporin
B Tacrolimus
C Rapamycin
D Corticosteroids
E Plasmapheresis
A

E Plasmapheresis

The patient in question has symptoms and signs suggestive of severe
myasthenia gravis (myasthenic crisis), typified by paralysis of the respiratory
muscles requiring ventilator assistance. The best treatment in this
scenario is plasmapheresis (E), a method of rapidly removing circulating
anti-acetylcholine receptor antibodies from the circulation; effects last
only for a short period. The patient’s own plasma is treated to remove
immunoglobulins, and then reinfused. Other indications for plasmapheresis
include Goodpasture’s syndrome (anti-glomerular basement membrane
proteins).
Pharmacological treatment of myasthenia gravis primary involves the
use of acetylcholinesterase inhibitors. However, immunosuppressive
agents, such as corticosteroids (D), also have a role. Corticosteroids
inhibit phospholipase A2, thereby blocking prostaglandin formation as
well as a spectrum of inflammatory mediators. The immunosuppressive
effects of corticosteroids are numerous and include reducing the number
of circulating B cells, inhibiting monocyte trafficking, inhibiting T-cell
proliferation and reducing the expression of a number of cytokines, for
example, IL-1, IL-2 and TNF-α. Prednisolone is used prophylactically
before transplantation to prevent rejection; methylprednisolone is used
in the treatment of rejection. Side effects are frequent, however, and
include osteoporosis, diabetes mellitus and hypertension.
Inhibitors of cell signalling which have been used in the management of
myasthenia gravis include ciclosporin (A). Ciclosporin inhibits the protein
phosphatase calcineurin. This in turn inhibits IL-2 secretion from
T cells, a cytokine which stimulates T-cell proliferation. Adverse effects
include nephrotoxicity, hepatotoxicity, diarrhoea and pancreatitis. Side
effects of cyclosporine use include gum hyperplasia. Other inhibitors of
cell signalling, although not indicated in myasethenia gravis management,
include tacrolimus (B) and rapamycin (sirolimus; C). Tacrolimus
is a calcineurin inhibitor that inhibits T-cell proliferation by binding to
FK-binding protein-1A (FKBP-1A), ultimately preventing T-cell activation.
Rapamycin inhibits T-cell proliferation by binding to FKBP-1A. Its
advantage lies in its low nephrotoxicity in comparison to other immunosuppressive
agents.

30
Q

A 56-year-old man who is due to undergo a kidney transplant is seen by the
transplant surgeon. The surgeon decides the patient should be started on an
immunosuppressive agent before the surgery to prevent rejection of the organ.
He prescribes a monoclonal antibody directed at the IL-2 receptor. Which drug
has been prescribed?

A Basiliximab
B Abatacept
C Rituximab
D Natalizumab
E Tocilizumab
A

A Basiliximab

Immunosuppressive agents which are directed against cell surface
antigens primarily target cluster of differentiation (CD) molecules.
Basiliximab (A) is an antibody directed towards IL-2 receptor α chain
(CD25) which causes reduction in T-cell proliferation. It is used as prophylactic
treatment of allograft rejection, most commonly in patients
undergoing kidney transplant. Adverse effects include increased risk of
infection as well as a long-term risk of malignancy. Abatacept (B) is a
CTLA4–immunoglobulin fusion protein indicated in the treatment of
rheumatoid arthritis, which has been resistant to treatment with disease
modifying drugs (DMARDs). Abatacept prevents antigen presenting cells
from delivering a co-stimulatory signal to T cells in order to promote
activation; this is achieved by abatacept binding with high affinity
to the B7 protein (CD80 and CD86) on the cell surface of APCs. Side
effects include increased risk of infection from TB, hepatitis B virus
and hepatitis C virus. Rituximab (C) is a CD20 monoclonal antibody
which causes reduced proliferation of B cells. It has a wide spectrum
of indications, including treatment of lymphoma, rheumatoid arthritis
and systemic lupus erythematosus. Adverse effects of rituximab include
increased risk of hepatitis B reactivation and progressive multifocal
leukoencephalopathy (PML). Natalizumab (D) is a monoclonal antibody
against α4-integrin, an adhesion receptor which mediates the migration
of T cells from the circulation to target organs; natalizumab prevents
this migration. It is used in the treatment of multiple sclerosis (reduced
T-cell migration to the central nervous system by influencing endothelial
cells expressing VCAM1) and Crohn’s disease (reduced interaction
of MADCAM1 and α4-integrin at sites of inflammation in the gastrointestinal
tract). Tocilizumab (E) is a monoclonal IL-6 receptor antibody,
indicated in Castleman’s disease and rheumatoid arthritis. IL-6 is a proinflammatory
cytokine which promotes the immune response; inhibition
thereby reduces macrophage, neutrophil, T-cell and B-cell activation.
Tocilizumab is hepatotoxic and raises serum cholesterol; liver function
tests and cholesterol must be monitored regularly.

31
Q

A 45-year-old woman who has been diagnosed with rheumatoid arthritis is seen
by a rheumatologist. The doctor wishes to start the patient on a fully humanized
TNF-α monoclonal antibody to prevent progression of the disease.

A Infliximab
B Adalimumab
C Etanercept
D Ustekinumab
E Denosumab
A

B Adalimumab

Immunosuppressive agents may be directed at specific cytokines to
modify the pathogenesis of certain disease processes. Adalimumab
(B) is a fully human monoclonal antibody to TNF-α. TNF-α has the
physiological role of inducing pro-inflammatory cytokines as well as
promoting leukocyte migration and endothelial adhesion. Adalimumab
has a large number of indications, including rheumatoid arthritis,
ankylosing spondylitis and Crohn’s disease. Infliximab (A) is a mouse–
human chimeric TNF-α antagonist indicated in similar conditions to
adalimumab. Infliximab has a high affinity for TNF-α. Toxicity may
result in reduced protection against infection from TB, hepatitis B virus
and hepatitis C virus, a lupus-like condition, demyelination and malignancy.
Etanercept (C) is also a TNF-α monoclonal antibody, which is
a fusion protein between the TNF-receptor 2 and Fc portion of IgG1.
Potential adverse effects include increased risk of infection, demyelination
and malignancy.
Ustekinumab (D) is an antibody to the p40 subunit of Il-12 and IL-23,
thereby preventing T-cell and natural-killer cell activation. It is used in
the treatment of psoriatic arthritis. Denosumab (E) is an antibody directed
towards the RANK ligand in bones. Osteoblasts are responsible for
bone formation, whilst osteoclasts (which contain the cell surface receptor
RANK) break down bone. Inhibition of RANK by denosumab therefore
inhibits osteoclast function and differentiation, thereby preventing
the breakdown of bone. Denosumab is indicated in the treatment of
osteoporosis but is also used in the management of multiple myeloma
and bone metastases. Toxicity can predispose to respiratory and urinary
tract infections.

32
Q

A 52-year-old woman presents to her GP with dry eyes and mouth for the past
few weeks. Despite using moisturizer the woman also complains of dry skin. The
patient has a history of coeliac disease. Which of the following antibodies is
most likely to be diagnostic for this patient’s condition?

A Anti-Jo1
B Anti-cyclic citrullinated protein
C Anti-centromere
D Anti-topoisomerase
E Anti-Ro
A

E Anti-Ro

This patient has presented with generalized dryness, a characteristic
clinical feature of Sjögren’s syndrome. Autoimmune destruction of the
epithelial cells of exocrine glands causes such features, including dryness
of the eyes (confirmed by Schirmer’s test) and mouth; other clinical
symptoms and signs include parotid swelling, fatigue, arthralgia and
myalgia. Anti-Ro (anti-SS-A; E) and Anti-La (anti-SS-B) antibodies are
present in approximately 50 per cent of patients with Sjögren’s syndrome,
as well as a lower proportion of patients with systemic lupus
erythematosus. Blood tests will demonstrate a raised ESR and occasionally
a mild anaemia. Anti-Jo1 (A) antibody is present in patients with
dermatomyositis. Dermatomyositis is characterized by autoimmune
inflammation of muscle fibres and skin. Clinical features include a heliotrope
rash around the eyes, Gottron’s papules on the dorsum of finger
joints, as well as weakness of the proximal limb muscles which causes
difficulty in climbing stairs and rising from a chair. Blood tests reveal
a raised creatine kinase level. Anti-cyclic citrullinated protein (antiCCP;
B) antibody is associated with rheumatoid arthritis. The antibody
is directed at the filament aggregating protein, filaggrin. Rheumatoid
arthritis is a chronic systemic autoimmune disease that results in a symmetrical
deforming polyarthritis. Clinical features include deformities
of the hands (Boutonierre’s deformity, swan-neck deformity, Z-thumb
and ulnar deviation of the fingers). Extra-articular manifestations
include pulmonary fibrosis, pericardial effusion, rheumatoid nodules
and splenomegaly (Felty’s syndrome). Anti-centromere (C) antibody is
associated with limited systemic scleroderma (CREST syndrome). CREST
syndrome is characterized by calcinosis, Reynaud’s syndrome, oesophageal
dysmotility, sclerodactyly and telangiectasia. Blood investigations
will reveal a raised ESR, anaemia and hypergammaglobulinaemia.
Anti-topoisomerase (D) antibody is characteristic of diffuse systemic
scleroderma. Diffuse systemic scleroderma shares some features of limited
systemic scleroderma, however, it is more aggressive in its course,
affecting large areas of the skin as well as involving the kidneys, heart
and lungs.

33
Q

A 42-year-old man is referred to the rheumatology outpatient clinic. The patient
has been experiencing muscle and joint pain for the past month. On examination
a heliotrope rash is observed on the patient’s eyelids. Blood tests reveal the
patient has circulating anti-nuclear antibodies. Which immunofluorescence staining
pattern will be observed in this disease process?

A Homogeneous
B Nucleolar
C Speckled
D Peripheral
E Kinetoplast
A

C Speckled

Anti-nuclear antibodies (ANA) are directed at the cell nucleus and are
present in a number of rheumatic autoimmune diseases. Indirect immunofluorescence
is an immunological technique that can be used to help
determine the ANA in question. In this scenario, the patient has signs
and symptoms suggestive of dermatomyositis. Dermatomyositis is characterized
by the presence of anti-Jo-1 antibodies, which will demonstrate
a speckled (C) pattern on immunofluorescence. Dermatomyositis
(and polymyositis) are inflammatory diseases of the peripheral skeletal
muscles. The disease is associated with HLA DR3 and DR52. Clinical
features include weakness of the proximal muscles of the arms and legs;
on direct questioning there may be difficulty climbing stairs for example.
Dermatological manifestations include the presence of a heliotrope
on the eyelids and Gottron’s papules. Dermatomyositis is associated
with increased risk of lung, ovary, breast and stomach cancer. Other
antibodies which demonstrate a speckled appearance on immunofluorescence
include anti-Smith (SLE), anti-RNP (mixed connective tissue
disease) and anti-Ro (Sjögren’s disease). A homogeneous pattern (A) is
consistent with the presence of anti-histone antibodies, characteristic of
drug-induced SLE. A nucleolar (B) pattern is indicative of anti-RNA polymerase,
which suggests underlying systemic sclerosis. A peripheral (D)
pattern on immunofluorescence is found in the presence of anti-double
stranded DNA (dsDNA) antibodies in SLE. Kinetoplasts (E) are the mitochondria
found in Crithidialuciliae, a non-pathogenic haemoflagellate,
and may be used as a substrate for pure dsDNA in the diagnosis of SLE.

34
Q

A 34-year-old woman, diagnosed with Chlamydia trachomatis infection 2 weeks
previously, sees her GP after experiencing a 1-week history of joint pain and
blurry vision. She also complains of a burning sensation when she passes urine.
Blood tests reveal a raised CRP and ESR. A joint aspirate of her knee is however
sterile. What is the most likely diagnosis?

A Ankylosing spondylitis
B Reactive arthritis
C Enteropathic arthritis
D Psoriatic arthritis
E Anterior uveitis
A

B Reactive arthritis

The seronegative spondyloarthritides are a collection of inflammatory
conditions which are rheumatoid factor negative. Other features
common to this group of diseases include the association with HLA
B27, involvement of the spine and sacroiliac joints and tendency to
enthesitis (inflammation at the site of attachment of tendons to bones).
This patient has symptoms, signs and investigative features suggestive
of reactive arthritis (B). The term ‘reactive’ in the disease name is
given due to the disease being preceded by infection (in this case by
Chlamydia trachomatis). Other predisposing infections include Shigella,
Yersinia and Campylobacter spp. Reactive arthritis is also known as
Reiter’s syndrome, which is defined by the triad of uveitis, urethritis and
arthritis (‘can’t see, pee or bend their knee’). The arthritis will typically
be transient and dactylitis may be a feature. Patients may also develop
constitutional symptoms such as fever, fatigue and weight loss. Blood
tests will reveal a raised ESR and CRP, while joint aspirate will be sterile.
Ankylosing spondylitis (AS; A) is a chronic inflammatory condition
involving the spine and sacroiliac joints; approximately 95 per cent of
patients are HLA B27 positive. It is believed the underlying autoimmune
pathogenesis occurs as a result of molecular mimicry. Characteristically,
stiffness of the joints is relieved by exercise. Acute iritis is common in
AS patients. Vertebral syndesmophytes are a feature on imaging of the
spine. Enteropathic arthritis (C) is associated with inflammatory bowel
disease. Psoriatic arthritis (D) occurs in conjunction with psoriasis, primarily
affecting the distal interphalyngeal joints and spine. Anterior
uveitis (E) causes redness of the eye, photophobia, excessive lacrimation
and blurred vision. It is often associated with other seronegative
spondylarthropathies.

35
Q

A 54-year-old woman is referred to a rheumatologist. The patient states that she
has noticed her fingers becoming very pale on cold days; when she heats her hands against the radiator, she notices her hands becoming red. She mentions
that she has also had joint pains in her hands. On inspection, the patient has a
small mouth. Which of the following factors is most responsible for fibrosis in
this disease process?

A von Willebrand factor
B IL-2
C TGF-β
D TNF-α
E Endothelin-1
A

C TGF-β

Systemic sclerosis is a chronic, inflammatory condition characterized
by fibrosis of the skin, blood vessels and internal organs. It can be
classified into a form that has major skin involvement (diffuse systemic
sclerosis) and a form in which skin involvement is limited to the distal
limbs and face (limited systemic sclerosis; CREST). CREST is defined by
calcinosis, Raynaud’s phenomenon, oesophageal dysmotility, sclerodactyly
and telangiectasia and is associated with the presence of circulating
anti-centromere antibodies. Given the absence of diffuse cutaneous
manifestations and combined with the symptoms and signs, the diagnosis
is limited systemic sclerosis. TNF-β (C) is central to the pathogenesis
of limited systemic sclerosis. Together with platelet-derived growth
factor (PDGF), TNF-β, produced by macrophages and T cells (IL-2 (B)
produced by CD4+ T cells induces further proliferation of T cells),
stimulate collagen production by fibroblasts. Collagen is deposited in
the extracellular matrix of the skin, oesophagus, alveoli of the lungs,
myocardium of the heart, liver and blood vessels; the pro-fibrotic state
correlates with the clinical features of limited systemic sclerosis.
In the early phase of the disease, activated T cells and TNF-α (D) cause
damage to endothelial cells. Endothelial disruption causes binding of
von Willebrand factor (vWF; A), with consequent platelet aggregation and release of PDGF. Endothelin-1 (E) is a potent vasoconstrictor, the
increased release of which contributes to Raynaud’s phenomenon as
well as systemic sclerosis-associated pulmonary hypertension.

36
Q

A 12-year-old boy is referred to the paediatric endocrinology outpatient clinic
after experiencing recent onset weight loss, tiredness, frequency of urination and
thirst. A fasting plasma glucose test reveal a level of 10.1mmol/L and a diagnosis
of type 1 diabetes mellitus is made. Which of the following autoantibodies
has tyrosine phosphatase as the target antigen?

A Islet cell surface antibody
B Insulin autoantibody
C Anti-glutamic acid decarboxylase antibody
D Anti-IA-2 antibody
E Islet cell antibody
A

D Anti-IA-2 antibody

Type 1 diabetes mellitus (T1DM) is a hyperglycaemic state caused by
autoimmune destruction of the β-cells in the islets of Langerhans of the
pancreas. The β-cells are responsible for the production of insulin. The
underlying pathogenesis of T1DM relates to T-cell mediated damage of
β-cells. The presence of glucose in the urine leads to the symptom of
polyuria (glucose is a potent osmolyte attracting water to enter the renal
tubules via osmosis). Polydipsia, weight loss and thirst are other characteristic
clinical features. An overnight fasting plasma glucose level of
above 7.0mmol/L is diagnostic of diabetes. Another investigative test
which can be used is the oral glucose tolerance test. T1DM affects men
and women equally and usually presents in the pubertal years. T1DM is
strongly associated with HLA DR3 and DR4 alleles.
A number of autoantibodies are implicated in the disease process
of T1DM. In this case autoantibodies to tyrosine phosphatase have
been detected. Two antibodies to tyrosine phosphatase are present in
T1DM: anti-IA-2 antibodies (D) and anti-phogrin antibodies. Tyrosine
phosphatase autoantibodies are found in approximately 75 per cent
of patients with T1DM. Islet cell surface antibodies (A) are found less
frequently in patients with T1DM. However, they are more specific for
pancreatic β-cells than tyrosine phosphatase autoantibodies and bind to
components of the surface of islet cells. Islet cell antibodies (ICA; E) are
directed to components of the islet cell cytoplasm. Patients with T1DM
may also have antibodies direct at insulin, or so called insulin autoantibodies
(B). Anti-glutamic acid decarboxylase antibody (anti-GAD
antibody; C) is not β-cell specific but is present in a high proportion
of T1DM patients. GAD is an enzyme responsible for the conversion
of glutamate to GABA; GABA is the neurotransmitter involved in the
release of insulin from β-cells.

37
Q

A 40-year-old woman presents to an endocrinologist with weight loss which
has occurred over the past month, associated with a tremor, excessive sweating
and a sense of feeling warm even on a cool day. On examination, the patient
has exophthalmos and an irregularly irregular pulse. Which of the following
autoantibodies is most likely to be responsible for the patient’s disease process?

A Anti-TSH receptor (stimulating)
B Anti-TSH receptor (non-stimulating)
C Anti-thyroid peroxidase
D Anti-thyroglobulin
E Thyroid growth stimulating antibody
A

A Anti-TSH receptor (stimulating)

Thyroid disease can be classified as hyperthyroidism (increased thyroid
activity) or hypothyroidism (reduced thyroid activity). Autoimmune
thyroid disease may be split into Graves’ disease (hyperthyroidism)
and Hashimoto’s thyroiditis (hypothyroidism), each with characteristic
clinical and immunological features. The patient in this scenario has
clinical features suggestive of Graves’ disease. Such symptoms include
unintentional weight loss, tremor, excessive sweating, heat intolerance,
palpitations and diarrhoea. Signs suggestive of Graves’ disease are a
goitre, proximal myopathy, brisk reflexes, tachycardia, atrial fibrillation,
pre-tibial myxoedema, ophthalmopathy and exophthalmos. Graves’
disease is defined by the presence of stimulating anti-TSH receptor (A)
antibodies, which bind to the TSH receptor, inducing production of
thyroxine. The pathophysiology behind exophthalmos is thought to be
due to similar receptors to TSH receptor in the extra-ocular muscles, to
which stimulatory anti-TSH antibodies bind, causing protrusion of the
eyes. Isotope scanning will demonstrate diffuse uptake. Thyroid growth
stimulating antibodies (E) are also found in patients with Graves’ disease
but to a far lesser extent. They induce growth of thyroid follicles.
Hashimoto’s thyroiditis is a chronic autoimmune disease, characterized
by thyroid under-activity. Features of hypothyroidism include weight
gain, cold intolerance, constipation, hoarse voice and menstrual abnormalities.
Signs indicative of hypothyroidism are slow reflexes, bradycardia
and an enlarged, nodular goitre. The two major antibodies found
in Hashimoto’s thyroiditis patients are anti-thyroid peroxidase (C) and
anti-thyroglobulin (D); thyroid peroxidase is an enzyme required in the
iodination of thyroglobulin. Non-stimulatory anti-TSH receptor (B) antibodies
are a feature of primary hypothyroidism.

38
Q

A 10-year-old boy is referred to a paediatrician after experiencing a seizure
1 week previously. Blood tests reveal that the seizure may have occurred
secondary to low calcium levels; blood glucose levels are found to be high.
The child was already being investigated for ptosis and difficulty with eye
movements. What is the most likely diagnosis?

A Hirata’s disease
B IPEX
C Kearns–Sayre syndrome
D POEMS syndrome
E APECED syndrome type 1
A

C Kearns–Sayre syndrome

The autoimmune polyendocrine syndromes are a group of conditions
characterized by autoimmune disease affecting numerous endocrine
(and non-endocrine) organs. This child has symptoms, signs and investigative
features consistent with Kearns–Sayre syndrome (oculocraniosomatic
disease; C). Kearns–Sayre syndrome is a myopathic disease
caused by deletions of mitochondrial DNA. Initially, the disease process
affects the eyelid and extra-ocular muscles leading to ptosis and difficulty
with eye movement. Pigmentary retinopathy is another feature,
causing diffuse pigmentation of the retina. Other clinical manifestations
of Kearns–Sayre syndrome are proximal muscle weakness, cardiac conduction
defects, hearing loss and cerebellar ataxia. Endocrine system
effects include: hypoparathyroidism (causing hypocalcaemia), primary
gonadal failure, diabetes mellitus and hypopituitarism. Hirata’s disease
(insulin autoimmune syndrome; A), in contrast to Kearne–Sayre syndrome,
is defined by fasting hypoglycaemia as well as autoantibodies to
serum insulin. It is most prevalent in Japan (third most common cause
of hypoglycaemia) but extremely rare in other countries. IPEX (B) is
otherwise known as immunodysregulation polyendocrinopathy enteropathy
X-linked syndrome and is caused by dysfunctional regulatory
T cells (as a result of abnormal FoxP3), ultimately predisposing to autoimmune
disease. The condition manifests with diabetes mellitus, eczema
and enteropathy. POEMS syndrome (D) is the acronym given to the
following collection of clinical features: polyneuropathy/papilledoema/
pulmonary disease, organomegaly/oedema, endocrinopathy, M-protein
(usually IgG or IgM) and skin abnormalities (hyperpigmentation and
hypertrichosis). APECED syndrome type 1 (autoimmune polyendocrine
syndrome type 1; E) is associated with mild immune deficiency, dysfunctional
parathyroid gland/adrenal gland, hypothyroidism, gonadal
failure, alopecia and vitiligo) and results from mutations in the AIRE
gene, a key player in central tolerance.

39
Q
A 6-year-old girl presents to accident and emergency with severe haematemesis,
endoscopy revealing the presence of oesophageal varices. Blood tests reveal liver
function test derangement and a low level of circulating IgA. Subsequent liver
biopsy demonstrates interface hepatitis. Treatment with steroids shows a poor
response. Which autoantibody is most likely to be present in this child?
A Anti-nuclear antibody
B Anti-smooth muscle antibody
C Anti-liver kidney microsomal antibody
D Anti-mitochondrial antibody
E Anti-HBs antibody
A

C Anti-liver kidney microsomal antibody

This patient is most likely to have autoimmune hepatitis (AIH) given the
biopsy findings of interface hepatitis, which is typical of the disease.
AIH is a disease of unknown aetiology characterized by inflammation,
hepatocellular necrosis and fibrosis, which may ultimately lead to cirrhosis
and liver failure. Diagnosis is based on a combination of histological
and antibody evidence. Patients will commonly have a history
of other autoimmune disease. In this case, the patient is most likely to
have AIH type 2 due to the early age of diagnosis (more common in
paediatric population) and poor steroid response. AIH type 2 is characterized
by the presence of anti-liver kidney microsomal antibodies (C).
AIH type 2 also has an association with IgA deficiency. A diagnosis of
AIH type 1 is suggested by the presence of anti-nuclear antibodies (A)
and anti-smooth muscle antibodies (B). AIH type 2 may be diagnosed
in patients from 10 years of age to elderly patients. The disease course
is less severe than type 2 and responds well to steroid therapy. There is
also a third type of AIH which is characterized by the presence of antisoluble
liver antigen antibodies.
Anti-mitochondrial antibodies (D) are present in patients with primary
biliary cirrhosis (PBC), a chronic liver disease characterized by the
destruction of intrahepatic bile ducts. Patients with PBC often have
raised IgM levels. The presence of anti-HBs (E) antibody would suggest
hepatitis B virus (HBV) infection. Anti-HBs antibodies are a marker of
acquired immune status.

40
Q

A 24-year-old man is referred to a gastroenterologist following episodes of diarrhoea
in the last month. The patient also feels more tired than usual. The man
undergoes a colonoscopy and jejunal biopsy results show villous hypertrophy
with crypt hyperplasia and an increase in intraepithelial lymphocytes. Which of
the following is associated with the greatest predisposition to developing this
disease?

A Dermatitis herpetiformis
B Vitiligo
C IgA deficiency
D HLA DQ8
E HLA DQ2
A

E HLA DQ2

Coeliac disease (gluten-induced enteropathy) is an autoimmune condition
triggered by gluten intolerance. It is specifically prolamins that
induce the immune response; examples of prolamins include gliadin
in wheat, hordein in barley and secalin in rye. It has been found
that it is the α-gliadin portion of gluten that is implicated in the
pathogenesis. Gliadin is resistant to degradation by gastrointestinal
enzymes and subsequently translocates across the intestinal barrier.
Within the lamina propria, gliadin is deamidated to glutamic acid via
the action of transglutaminase. This creates a negative charge which,
in genetically susceptible individuals, is able to bind to HLA DQ2 (E)
on the surface of antigen presenting cells with high affinity. Patients
with the HLA DQ8 (D) allele are also susceptible to developing coeliac
disease but to a far lesser extent. The peptide is presented to T-helper
cells which activate plasma cells to produce antibodies (anti-gliadin,
anti-endomysial and anti-tissue transglutaminase antibodies), as well
as lymphocytes such as natural-killer cells and macrophages, which in
turn release IFN-γ, IL-4 and TNF-α causing enterocyte damage. Disease
is characterized histologically by villous atrophy, crypt hyperplasia and
an intraepithelial infiltrate of lymphocytes. Clinical features include
abdominal pain, diarrhoea, steatorrhoea, as well as symptoms and signs
of nutrient deficiencies.
Coeliac disease is also strongly associated with dermatitis herpetiformis
(A), a dermatological condition characterized by sub-epidermal deposits
of IgA. Coeliac disease may exist in conjunction with other autoimmune
diseases such as vitiligo (B), systemic lupus erythematosus and/or type 1
diabetes mellitus. Patients with IgA deficiency are also at increased risk
of developing coeliac disease (C) compared to those who are not deficient;
IgA deficiency increases the difficulty of serological testing.

41
Q

A 52-year-old Mediterranean woman is referred to the dermatology outpatient
clinic as a result of blisters that have developed in her mouth and on her arms.
The patient describes the blisters as being very fragile and rupturing easily.
Immunological testing reveals the presence of anti-desmoglein 3 antibodies and
punch biopsy of a lesion demonstrates the presence of acantholytic cells. What is
the most likely diagnosis?

A Pemphigus foliaceous
B Pemphigus vulgaris
C Bullous pemphigoid
D Epidermolysis bullosa
E Dermatitis herpetiformis
A

B Pemphigus vulgaris

Autoimmune skin disorders are characterized by autoantibodies directed
at components of the epidermis, basement membrane zone and dermis.
This patient has presented with a blistering condition (bullous disease).
The fact that anti-desmoglein 3 antibodies have been detected, together
with the histological finding of acantholysis, point to pemphigus
vulgaris (B) as the correct diagnosis. Pemphigus vulgaris is more prevalent
in the Mediterranean population. Direct immunofluorescence reveals
intercellular epidermal IgG and C3 deposition, while indirect immunofluorescence
demonstrates intercellular IgG. Skin histology shows the
presence of acantholytic cells, which is defined as the separation of
keratinocytes caused by loss of intercellular cadherin connections.
Clinical features include blisters appearing in the mouth and skin, which
are very friable. Unaffected skin becomes increasingly fragile and exfoliation
of such areas occurs with light rubbing (Nikolsky sign positive).
High dose steroids (with or without immunosuppressive agents such as
azathioprine) is the mainstay treatment.
Pemphigus foliaceous (A) is characterized by immunological and histological
findings similar to pemphigus vulgaris. However, it is desmoglein
1 which is the target for autoantibodies, and the clinical course is
far less severe in the case of pemphigus foliaceous. Autoantibodies to
components of hemidesmosomes of the basement membrane zone are
associated with bullous pemphigoid (C). Indirect immunofluorescence
demonstrates a linear pattern of IgG and C3 at the basement membrane
zone. Clinical features include generalized blisters on the skin which are
very itchy. Epidermolysis bullosa (D) is caused by autoantibodies to type
VII collagen, which forms anchors between the layers of the skin; as a
result, bullae are usually induced by trauma. Dermatitis herpetiformis (A)
is characterized by vesicles located on extensor surfaces. Immunological
studies reveal deposits of IgA at the dermal papillae. Dermatitis herpetiformis
is associated with coeliac disease, and anti-gliadin and antiendomysial
antibodies may be present in the circulation.

42
Q

An Afro-Caribbean man with a history of type 1 diabetes mellitus presents to the
dermatology outpatient clinic with depigmented areas of his face, arms and legs.
On examination the affected areas are completely white. The patient admits that
the lesions are leading to low mood. Which of the following is most associated
with this disease process?

A	 β-Haemolytic streptococcal infection
B Vancomycin
C Pregnancy
D Anti-melanocyte antibodies
E Multiple myeloma
A

D Anti-melanocyte antibodies

This patient has presented with skin depigmentation, lesions appearing
completely white; the most likely diagnosis is vitiligo. It is associated
with autoimmune disease, including type 1 diabetes, pernicious anaemia
and Addison’s disease. Vitiligo is also a common feature in patients
with autoimmune polyendocrine syndromes. The pathogenesis of vitiligo
involves the autoimmune destruction of melanocytes, which are
responsible for the production of the pigment melanin; anti-melanocyte
antibodies (D) are found in patients with vitiligo. As a result, histology
of affected areas will reveal an absence of melanocytes. Other causes of
hypopigmentation (which are usually generalized) include albinism and
phenylketonuria.
Psoriasis is a chronic skin condition characterized by pink/salmon
plaques covered in silvery scales which occur primarily over the extensor
surfaces of the body as well as the scalp. Other clinical features
include pitting of the nail beds and onycholysis. Auspitz’s sign relates
to the disease and is defined by pin-point bleeding when psoriatic
plaques are peeled away. Triggers for disease are thought to include
β-haemolytic streptococcal infection (A), trauma, drugs and UV radiation.
The pathogenesis of psoriasis involves the migration of neutrophils
and T cells from the dermis to the epidermis, which induce skin
cell proliferation. Vancomycin (B) is associated with linear IgA bullous
dermatosis (LABD), characterized by linear deposition of IgA on direct
immunofluorescence. Pemphigoid gestationis is a bullous disorder associated
with pregnancy (C). Bullae appear in the second or third trimester
of pregnancy, which are characterized by itchiness; the condition tends
to resolve post-partum. Multiple myeloma (E), as well as other lymphoproliferative
disorders, is associated with paraneoplastic pemphigus.

43
Q

A 4-year-old boy is referred to a renal physician after his mother noticed swelling
of his legs. A week previously the boy had been stung by a bee. Urine dipstick
reveals the presence of proteinuria, while blood tests show hypoalbuminaemia
and hyperlipidaemia. The child’s symptoms rapidly disappear with a
course of steroids. What is the most likely diagnosis?

A Alport syndrome
B Reflux nephropathy
C Shunt nephritis
D Systemic lupus erythematosus
E Minimal change disease
A

E Minimal change disease

The non-proliferative glomerulonephritides are characterized by a lack
of hypercellularity of the glomeruli. This group of conditions cause
nephrotic syndrome, as this child has presented with, defined by the
classical triad of hypoalbuminaemia, proteinuria (greater than 3.5g) and
oedema. Hyperlipidaemia and lipiduria may also be present. The pathogenesis
of nephrotic syndrome begins with immune damage to the glomerulus
which subsequently becomes leaky; proteinuria arises leading
to hypoalbuminaemia. Hypoalbuminaemia consequently causes reduction
in oncotic pressure and hence oedema ensues. In an attempt to
maintain oncotic pressure the liver compensates by increasing lipid production;
those are lost through the leaky glomeruli (lipiduria). The most
common cause of nephrotic syndrome in children is minimal change
disease (E), which is the correct answer in this scenario due to the
recent allergic reaction (bee sting; type I hypersensitivity reaction) as
well as the strong response to steroid treatment, which leads to remission
of disease in the vast majority of cases. Histological characteristics
of renal biopsy specimens include a lack of structural change visible on
light microscopy, while electron microscopy will demonstrate podocyte
effacement.
Focal segmental glomerulonephritis (FSGN) is another cause of nephrotic
syndrome, characterized histologically by focal sclerosis of glomeruli.
Immunofluorescence will reveal the presence of IgM and C3 deposition
in affected areas. Patients will usually present with some degree of renal
impairment. FSGN may be idiopathic or occur secondary to conditions
such as Alport syndrome (A) and reflux nephropathy (B). Alport syndrome
is a hereditary syndrome (mutation of α4 chain of type IV collagen)
associated with glomerulonephritis, end-stage kidney disease and
hearing loss. Reflux nephropathy results from vesico-ureteric reflux due
to chronic pyelonephritis.
Membranous glomerulonephritis most commonly occurs in adults,
and demonstrates a thickened glomerular basement membrane and
spike/dome protrusions on histology. Direct immunofluorescence
reveals the presence of sub-epithelial granular deposits of IgG and
C3. Causes of membranous glomerulonephritis include infections,
neoplasia, drugs and connective tissue disease, for example systemic
lupus erythematosus (D). Patients with hydrocephalus who have a
cerebral shunt in situ are prone to shunt nephritis (C), a cause of
membranous glomerulonephritis. The pathogenesis involves the
increased risk of long-term bacterial infection, leading to immune
complex deposition in the glomeruli.

44
Q

A 24-year-old woman is seen by the GP after noticing she is urinating less often
as well as seeing some blood when she does pass water. Urine investigations
reveal the presence of red cell casts and dysmorphic red blood cells. The patient
admits to having had a sore throat 2 weeks previously. Anti-streptolysin O titres
are raised. What is the most likely diagnosis?

A IgA nephropathy
B Henoch–Schonlein purpura
C Post-streptococcal glomerulonephritis
D Membranoproliferative glomerulonephritis
E Rapidly progressive glomerulonephritis
A

C Post-streptococcal glomerulonephritis

Proliferative glomerulonephritides is characterized by an increased
number of cells in the glomerulus. This group of diseases usually
present with nephritic syndrome, defined by the presence of haematuria,
red cell casts, dysmorphic red cells, oliguria and hypertension.
Proteinuria and oedema may also be present. Immune damage to
the glomerular vessels results in severe inflammation, allowing red
cells to pass into the tubule; in the process these red cells experience
mechanical damage while passing through the inflamed vessels and
as a result are dysmorphic. Cells of the distal convoluted tubule and
collecting duct secrete a glycoprotein called Tamm–Horsefell protein
which sticks red cells together forming cylindrical red cell casts. The
patient in this scenario has features of nephrotic syndrome; the history
of a sore throat preceding the new symptoms, combined with
the raised anti-streptolysin O titre, suggests a streptococcal infection.
Post-streptococcal glomerulonephritis (C) is usually caused by a preceding
infection (most commonly group A β haemolytic streptococci).
Pathological hallmarks of post-streptococcal glomerulonephritis include
diffuse hypercellularity and diffuse swelling of the mesangium and
glomerular capillaries. Direct immunofluorescence reveals the subepithelial
deposition of IgG and C3. The condition usually subsides
with supportive treatment.
IgA nephropathy (Berger’s disease; A) is the most common cause of
glomerulonephritis in the developed world. Characteristically there is
mesangial deposition of IgA. The condition occurs a few days after
a gastrointestinal or upper respiratory infection, especially caused by
Haemophilus influenzae. Henoch–Schönlein purpura (HSP; B) is a small
vessel vasculitis in which IgA and C3 are deposited in blood vessels,
leading to systemic clinical effects. In contrast to IgA nephropathy,
which tends to affect adults, HSP has a greater prevalence in children.
Membranoproliferative glomerulonephritis (D) is defined by mesangial
cell proliferation with thickening of the capillaries. Two types exist:
type 1 in which there is classical and alternative complement pathway
activation and type 2 which is associated with only alternative pathway
activation.
Rapid progressive glomerulonephritis (RPGN; E) is the most aggressive
of all glomerulonephritides, and may cause end-stage renal failure over
a period of days. The three sub-types include immune complex disease,
pauci-immune disease and anti-glomerular basement membrane disease,
all of which demonstrate the crescent sign on biopsy (proliferation of
macrophages and parietal epithelial cells).

45
Q

A 44-year-old man with known systemic lupus erythematosus is seen by a renal
physician. Initially the patient had proteinuria on a routine urine dipstick. A
subsequent renal biopsy demonstrated granular patterned deposition of IgG, IgM,
IgA and C3 confined to the mesangium on both light and electron microscopy.
Which stage of lupus nephritis is suggested by these findings?

A Stage I
B Stage II
C Stage III
D Stage IV
E Stage V
A

B Stage II

Systemic lupus erythematosus (SLE) is a multisystem autoimmune
condition characterized by the presence of anti-nuclear antibodies;
glomerunephritis is a common complication of the disease process, also
known as lupus nephritis. Generally, lupus nephritis is characterized by
the deposition of IgG, IgM, IgA and C3 in the sub-endothelial segment
of the glomerular basement membrane and in the mesangium. However,
the disease can be more accurately classified based on the pathological
features, which is a useful tool for monitoring disease and assessing
severity. The classification is as follows:
Stage Light microscopy
Electron microscopy and
immunofluorescence
Stage I (minimal mesangial
lupus nephritis)
No changes to glomeruli Mesangial immune deposits
Stage II (mesangial
proliferative lupus nephritis)
Changes confined to
mesangium
Mesangial immune deposits
Stage III (focal lupus
nephritis)
Focal, segmental or
glomerulonephritis involving
<50 per cent of all glomeruli
Subendothelial and mesangial
immune deposits
Stage IV (diffuse proliferative
nephritis)
Focal, segmental or
glomerulonephritis involving
>50 per cent of all glomeruli
Subendothelial immune deposits
Stage V (membranous
lupus nephritis)
Glomerular sclerosis involving
>90 per cent of glomeruli,
fibrosis and tubular atrophy
Subepithelial and
intramembranous immune
deposits
From the above classification, it is clear that the biopsy investigation
points to stage II (B) as the correct answer. The stage of disease is also
consistent with the clinical features. Stage II suggests mild disease with
haematuria or proteinuria; supportive treatment is warranted. Stage I
(A) is characterized by mild proteinuria. Stage III (C) suggests mild-tomoderate
disease which may be accompanied by worsening renal function
tests. Stage IV (D) suggests moderate–severe disease; the patient
may have hypertension, reflecting renal disease, worsening renal function
tests and active SLE. Stage V (E) is characterized by the presence of
nephrotic syndrome.

46
Q

A 53-year-old man presents to accident and emergency with haemoptysis. Blood
tests demonstrate deranged renal function and further tests reveal the presence
of circulating c-ANCA antibodies. The patient is noted to have a saddle-shaped
nose. What is the most likely diagnosis?

A Cryoglobulinaemia
B Wegener’s granulomatosis
C Microscopic polyarteritis
D Polyarteritis nodosa
E Churg–Strauss syndrome
A

B Wegener’s granulomatosis

The vasculitides are a group of conditions that are characterized by
inflammation within blood vessels leading to systemic clinical manifestations.
The vasculitides are classified based on the size of vessel affected;
renal disease is usually caused by either small or intermediate vessel
vasculitides. The patient in this scenario has presented with epistaxis,
haemoptysis and is positive for cytoplasmic anti-neutrophil cytoplasmic
antibody (c-ANCA); these features point to Wegener’s granulomatosis as
the correct answer (B). The antibody, c-ANCA, is central to the pathogenesis,
and is directed towards proteinase 3 (PR3), an enzyme normally
present within the cytoplasm of neutrophils. It is proposed that an infection
is the trigger for the disease, which causes circulating neutrophils
to become adherent to the endothelium; in the process PR3 is upregulated
on the neutrophil cell surface as well as being released into the
blood vessel lumen. c-ANCA present in the circulation binds to the cell
surface PR3 on neutrophils preventing them from migrating through
the endothelium. Vasculitis is caused by both direct effect of PR3 on
the endothelium as well as c-ANCA–PR3 immune complex deposition.
Cryoglobulinaemia (A) is defined by the presence of cryoglobulins in
the circulation; these are immunoglobulins that precipitate at low
temperatures. Secondary causes include connective tissue diseases and
lymphoproliferative conditions. It is, however, unknown why such
immunoglobulins are formed in the first instance. When precipitation
does occur at cold temperatures, the immunoglobulins adhere to vessel
walls, leading to complement activation, neutrophil recruitment and,
consequently, vessel damage. Microscopic polyangitis (C) is a small
vessel vasculitis affecting the arterioles, venules and capillaries. This
vasculitis is associated with focal necrotizing glomerulonephritis as well
as the presence of perinuclear ANCA in the circulation (directed towards
cytoplasmic myeloperoxidase). Polyarteritis nodosa (PAN; D) is a vasculitis
of small and medium-sized vessels associated with hepatitis B infection.
Immune complexes (type III hypersensitivity reaction) are deposited
within such vessels leading to fibrinoid necrosis and neutrophil infiltration;
as a result the vessel walls weaken and there is aneurysm development.
PAN is associated with the presence of p-ANCA antibodies.
Angiogram will reveal multiple aneurysms. Churg–Strauss syndrome (E)
is a medium-and small-vessel autoimmune vasculitis. Blood vessels of
the lungs, gastrointestinal system and peripheral nerves are most commonly
affected. There is an association with p-ANCA antibodies.

47
Q

A 35-year-old builder is referred to a neurologist after experiencing increasing
axial rigidity over the previous few weeks; his symptoms are interfering
with his work. The patient has a history of type 1 diabetes mellitus and vitiligo.
Immunological investigations reveal the presence of circulating anti-glutamic
acid decarboxylase antibodies. What is the most likely diagnosis?

A Myasthenia gravis
B Multiple sclerosis
C Acute disseminated encephalomyelitis
D Lambert–Eaton myasthenic syndrome
E Stiff man syndrome
A

E Stiff man syndrome

The patient in question has presented with axial rigidity/stiffness associated
with a history of autoimmune disease and circulating anti-glutamic
acid decarboxylase antibodies (anti-GAD), which point to stiff man
syndrome (SMS; E) as the correct answer. SMS is a very rare neurological
condition which is poorly understood. Clinical features include
progressive axial and abdominal wall stiffness. It is strongly associated
with the presence of anti-GAD antibodies. However, only a small
minority of type 1 diabetes mellitus patients suffer with SMS, suggesting
that anti-GAD antibodies do not tell the whole story in terms of
aetiology. However, SMS does occur in patients who suffer from other
autoimmune diseases including thyroid disease, pernicious anaemia
and type 1 diabetes mellitus. Myasthenia gravis (A) is characterized by
autoantibodies directed towards the acetylcholine receptor located at the
neuromuscular junction. Myasthenia gravis is a condition which presents
with fatigability of muscles; muscles become fatigued after periods of
movement but recover after rest. In severe cases, muscles of breathing
may become affected, leading to respiratory distress. The presence of a
thymus and treatment with D-penicillamine for rheumatoid arthritis are
associated with the development of myasthenia gravis. Lambert–Eaton
myasthenic syndrome (LEMS; D) is defined by proximal muscle weakness,
which is improved by muscle contraction, loss of tendon reflexes
and autonomic nervous system dysfunction. Leg involvement is greater
than that of myasthenia gravis. It is considered a paraneoplastic syndrome
due to its association with small cell lung cancer. LEMS is caused
by autoantibodies that target the voltage-gated calcium channels of the
pre-synaptic membrane. The pathogenesis of multiple sclerosis (MS; B)
is mediated by T cells. Proteolipid protein and myelin basic protein are
oligodendrocyte proteins implicated in the pathogenesis of MS.
MS is a demyelinating disease in which the myelin sheaths surrounding
neurons of the brain and spinal cord are destroyed. Clinical features
of MS include optic neuritis, urinary/bowel incontinence, weakness
of the arms/legs and dysphagia. Uhthoff’s phenomenon describes the
worsening of symptoms that occurs after exposure to higher than ambient
temperatures. Acute disseminated encephalomyelitis (ADEM; C) is a
demyelinating condition that follows vaccination or infection. Clinical
features include fever, headache and reduced consciousness; focal signs
include optic neuritis, cranial nerve palsies and seizures. Most cases are
followed by recovery within a few months.

48
Q

A 35-year-old man is transferred to the intensive care unit for ventilator support
after suffering an episode of respiratory distress. The patient was admitted 5 days
previously after experiencing weakness of his legs. Approximately 2 weeks prior
to his admission the man had suffered a bout of gastroenteritis caused by the
bacterium Campylobacter jejuni. Which of the following is the most likely antigenic
target for autoantibodies in this disease process?

A Ganglioside LM1
B Ganglioside GM1
C Hu
D Myelin-associated glycoprotein
E Purkinje cells
A

A Ganglioside LM1

Several polyneuropathies have an underlying immune component,
characterized by the presence of autoantibodies targeted at components
of the nervous system. In this scenario, the patient has experienced
weakness following a gastrointestinal infection, now complicated by
respiratory involvement. The most likely diagnosis is Guillain–Barrè
syndrome (GBS), for which ganglioside LM1(A) is the implicated target
for autoantibodies. GBS is a symmetrical inflammatory polyneuropathy
that begins in the legs and ascends to involve motor neurons of the
arms, face and finally those supplying muscles of respiration. GBS usually
follows an infection, most frequently after viral infection such as
cytomegalovirus or bacterial gastroenteritis caused by Campylobacter
jejuni. The pathogenesis involves cross-reactivity between antibodies
against the pathogen and components of peripheral nerve myelin
components, such as ganglioside LM1. Other potential myelin targets
include P2 protein and galactocerebroside. Amyotrophic lateral sclerosis
(ALS) is a sub-type of motor neuron disease characterized by loss
of neurons in the motor cortex as well as anterior horn of the spinal
cord; it is therefore associated with both upper and lower motor signs.
The pathogenesis of ALS has been suggested to be due to antibodies to
the ganglioside GM1 (B). Paraneoplastic subacute sensory neuropathy
(PSSN) is associated with malignancies such as small cell lung cancer.
Antibodies are directed at Hu (C) proteins which are a constituent part
of peripheral nerves. Paraneoplastic cerebellar degeneration is associated
with antibodies to Purkinje cells (E) of the central nervous system.
The pathogenesis is thought to be secondary to cross-reactivity between
antibodies to tumour cells and antigens present on cerebellar Purkinje
cells. Paraprotein-associated polyneuropathy is typified by the presence
of antibodies that target myelin-associated glycoprotein (MAG; D)

49
Q

A 35-year-old woman is referred to an ophthalmologist after seeing floaters in
her right eye. On examination, there is loss of accommodation in the same eye.
The patient’s notes reveal there had been trauma to the left eye following a car
accident 3 weeks previously. It is explained to the patient that she could suffer
potential loss of vision if steroid treatment is not commenced urgently. What is
the most likely diagnosis?

A Keratoconjunctivitis sicca
B Sympathetic ophthalmia
C Uveitis
D Keratitis
E Scleritis
A

B Sympathetic ophthalmia

Immune disorders of the eye can be classified according to the anatomical
site of disease: cornea, sclera/episclera, uvea and retina. This
patient presents with floaters and loss of accommodation in her right
eye, several weeks after experiencing trauma to her left eye. The most
likely diagnosis is therefore sympathetic ophthalmia (B), a granulomatous
CD4+ T-cell mediated disease. The trigger for the disease is trauma
to the damaged eye. The eye is an immunoprivileged site and is therefore,
under normal circumstances, protected from possible autoimmune
attack. Trauma to the eye breaks such tolerance, and there is consequently
increased photoreceptor antigen presentation to immune cells,
triggering cytokine release and recruitment of CD4+ T cells. These CD4+
cells soon encounter the same antigen presented at normal levels in
the healthy eye, leading to a break in tolerance. Activated T cells cause
ocular damage which may, in severe cases, lead to blindness.
Immune diseases of the cornea include keratoconjunctivits sicca (A) and
keratitis (D). Keratoconjunctivits sicca is a feature of Sjögren’s syndrome
when present with a dry mouth and rheumatoid arthritis. It is caused by
autoimmune destruction of the lacrimal glands, which leads to dry eyes
which may predispose to infection. Schirmer test can be used to formally
diagnose the condition. Keratitis may be caused by herpes simplex
viral infection; this leads to recurrent bouts of keratitis.
Diseases of the sclera include scleritis (E), which occurs as a result of
chronic inflammatory conditions such as connective tissue diseases
(ankylosing spondylitis, systemic lupus erythematosus and rheumatoid
arthritis), as well as type IV hypersensitivity reactions. Episcleritis is also
an immune disease of the sclera which is a self-limiting condition.
Uveitis (C) is defined as inflammation of the uveal tract, which occurs
either as an idiopathic disease, or secondary to chronic inflammatory
conditions as with scleritis, including inflammatory bowel disease

50
Q

A 52-year-old woman diagnosed with systemic lupus erythematosus develops
jaundice and on examination is found to have conjunctival pallor. Blood tests
reveal an elevated unconjugated bilirubin. Which of the following is the most
useful investigation to determine the diagnosis?

A Skin prick test
B Direct antiglobulin test
C Western blot
D Immunofluorescence test
E Patch testing
A

B Direct antiglobulin test

The patient in question has signs and blood test results consistent
with haemolysis; together with the known diagnosis of systemic lupus
erythematosus (SLE), this points to autoimmune haemolytic anaemia
(AIHA) as a likely diagnosis. AIHA is caused by autoantibodies that
bind to red blood cells (RBCs) leading to destruction in the spleen. AIHA
can be classified as either ‘warm’ or ‘cold’ depending on the temperature
at which antibodies bind to RBCs. Warm AIHA is IgG mediated,
which bind to RBCs at 37°C; causes include lymphoproliferative disorders,
drugs (penicillin) and autoimmune diseases (SLE). Cold AIHA is
IgM mediated, which bind to RBCs at temperatures less than 4°C; this
phenomenon usually occurs after an infection by mycoplasma or EBV.
Direct antiglobulin test (DAT; B), also known as direct Coombs test, is
the investigation of choice for the diagnosis of AIHA. The test involves
the separation of RBCs from the serum which is subsequently incubated
with anti-human globulin. In the case of AIHA, the anti-human globulin
will agglutinate the RBCs, which is visualized as clumping of the
cells. Skin prick test (A) is the gold standard for investigating allergy.
The test involves a few drops of purified allergen being pricked onto
the skin. Allergens which are tested for include foods, dust mites, pollen
and dust. A positive test is indicated by wheal formation. Western blot
(C) is a technique used to detect specific protein in a patient’s serum; it
is used in the confirmatory HIV test to detect specific antibodies to HIV.
Immunofluorescence (D) is an immunological technique used in conjunction
with fluorescence microscopy. Fluorophores (fluorescent chemical
compounds) attached to specific antibodies are directed at antigens
found within a biological specimen, most commonly a biopsy sample, to
visualize patterns of staining. Patch testing (E) is a useful test to determine
the causative allergen in contact dermatitis. A patch is prepared
with small amounts of allergens; a positive test may be demonstrated by
a spectrum of responses, from faint erythema to the presence of bullae.